You are on page 1of 80

LIALMC07_0321227638.

QXP

2/26/04

10:47 AM

Page 605

Trigonometric Identities and Equations


In 1831 Michael Faraday discovered that
when a wire passes by a magnet, a small electric current is produced in the wire. Now we generate massive amounts of electricity by simultaneously rotating thousands of wires near large electromagnets. Because electric current alternates its direction on electrical wires, it is modeled accurately by either the sine or the cosine function. We give many examples of applications of the trigonometric functions to electricity and other phenomena in the examples and exercises in this chapter, including a model of the wattage consumption of a toaster in Section 7.4, Example 5.

7.1 Fundamental Identities 7.2 Verifying Trigonometric Identities 7.3 Sum and Difference Identities 7.4 Double-Angle Identities and HalfAngle Identities Summary Exercises on Verifying Trigonometric Identities

7.5 Inverse Circular Functions 7.6 Trigonometric Equations 7.7 Equations Involving Inverse
Trigonometric Functions

605

LIALMC07_0321227638.QXP

2/26/04

10:47 AM

Page 606

606 CHAPTER 7 Trigonometric Identities and Equations

7.1 Fundamental Identities


Negative-Angle Identities
y
s

Fundamental Identities

Using the Fundamental Identities

(x, y) y y r (x, y) y sin( ) = r = sin


Figure 1

r x O
x

Negative-Angle Identities As suggested by the circle shown in Figure 1, an angle having the point x, y on its terminal side has a corresponding angle with the point x, y on its terminal side. From the denition of sine,
sin y r and sin y , r
(Section 5.2)

so sin and sin are negatives of each other, or sin sin . Figure 1 shows an angle in quadrant II, but the same result holds for in any quadrant. Also, by denition, cos so x r and cos x , r
(Section 5.2)

cos cos .

We can use these identities for sin and cos to nd tan in terms of tan :
TEACHING TIP Point out that in
trigonometric identities, can be an angle in degrees, a real number, or a variable.

tan

sin sin sin cos cos cos tan tan .

Similar reasoning gives the following identities. csc csc , sec sec , cot cot

This group of identities is known as the negative-angle or negative-number identities.

Fundamental Identities In Chapter 5 we used the denitions of the trigonometric functions to derive the reciprocal, quotient, and Pythagorean identities. Together with the negative-angle identities, these are called the fundamental identities.

Fundamental Identities
Reciprocal Identities cot Quotient Identities tan sin cos cot cos sin
(continued)

1 tan

sec

1 cos

csc

1 sin

LIALMC07_0321227638.QXP

2/26/04

10:47 AM

Page 607

7.1 Fundamental Identities

607

TEACHING TIP Encourage students


to memorize the identities presented in this section as well as subsequent sections. Point out that numerical values can be used to help check whether or not an identity was recalled correctly.

Pythagorean Identities sin2 cos2 1 Negative-Angle Identities sin() sin csc() csc cos() cos sec() sec tan() tan cot() cot tan2 1 sec2 1 cot2 csc2

NOTE The most commonly recognized forms of the fundamental identities are given above. Throughout this chapter you must also recognize alternative forms of these identities. For example, two other forms of sin2 cos2 1 are

sin2 1 cos2

and

cos2 1 sin2 .

Using the Fundamental Identities One way we use these identities is to find the values of other trigonometric functions from the value of a given trigonometric function. Although we could nd such values using a right triangle, this is a good way to practice using the fundamental identities.

EXAMPLE 1 Finding Trigonometric Function Values Given One Value and the Quadrant
5 If tan 3 and is in quadrant II, nd each function value.

(a) sec
Solution

(b) sin

(c) cot

(a) Look for an identity that relates tangent and secant.


5 3
TEACHING TIP Warn students that
the given information in 5 5 Example 1, tan , 3 3 does not mean that sin 5 and cos 3. Ask them why these values cannot be correct.

tan2 1 sec2
2

Pythagorean identity tan 5 3

1 sec2

25 1 sec2 9

34 sec2 9 34 sec 9

Combine terms.

Take the negative square root. (Section 1.4)

34 sec 3

Simplify the radical. (Section R.7)

We chose the negative square root since sec is negative in quadrant II.

LIALMC07_0321227638.QXP

2/26/04

10:47 AM

Page 608

608 CHAPTER 7 Trigonometric Identities and Equations

(b)

tan

sin cos

Quotient identity Multiply by cos . Reciprocal identity


1 3 334 From part (a), sec 34 34 ; 5 tan 3


334 34 5 3 (c) cot cot cot

cos tan sin

1 tan sin sec sin 534 34

sin

1 tan 1 tan 1 3 5 3 5

Reciprocal identity

Negative-angle identity tan 5 3 ; Simplify. (Section R.5) Now try Exercises 5, 7, and 9.

To avoid a common error, when taking the square root, be sure to choose the sign based on the quadrant of and the function being evaluated.
CAUTION

Any trigonometric function of a number or angle can be expressed in terms of any other function.
EXAMPLE 2 Expressing One Function in Terms of Another

Express cos x in terms of tan x.


Solution

Since sec x is related to both cos x and tan x by identities, start with 1 tan2 x sec2 x. 1 1 1 tan2 x sec2 x
Take reciprocals.

1 cos2 x 1 tan2 x 1 cos x 1 tan2 x cos x cos x 1 1 tan2 x

Reciprocal identity

Take square roots. Quotient rule (Section R.7); rewrite.

1 tan2 x Rationalize the denominator. (Section R.7) 1 tan2 x

Choose the sign or the sign, depending on the quadrant of x.


Now try Exercise 43.

LIALMC07_0321227638.QXP

2/26/04

10:47 AM

Page 609

7.1 Fundamental Identities

609

We can use a graphing calculator to decide whether two functions are identical. See Figure 2, which supports the identity sin2 x cos2 x 1. With an identity, you should see no difference in the two graphs. s All other trigonometric functions can easily be expressed in terms of sin and/or cos . We often make such substitutions in an expression to simplify it.
Y1 = Y2
4

EXAMPLE 3 Rewriting an Expression in Terms of Sine and Cosine

Write tan cot in terms of sin and cos , and then simplify the expression.
Solution

4
Figure 2

tan cot

sin cos cos sin

Quotient identities

y2 =

y1 = tan x + cot x 1 cos x sin x


4

sin2 cos2 Write each fraction with the cos sin cos sin LCD. (Section R.5) sin2 cos2 cos sin 1 cos sin
Add fractions.

tan cot

Pythagorean identity

Now try Exercise 55.

4 The graph supports the result in Example 3. The graphs of y1 and y2 appear to be identical.

CAUTION When working with trigonometric expressions and identities, be sure to write the argument of the function. For example, we would not write sin2 cos2 1; an argument such as is necessary in this identity.

7.1 Exercises
1. 2.6 2. .65 3. .625 310 7 4. .75 5. 6. 4 10 25 7. 5 9. 105 11 77 8. 11 10. 5 8

Concept Check

Fill in the blanks. . . . .

1. If tan x 2.6, then tanx 2. If cos x .65, then cosx 3. If tan x 1.6, then cot x

4. If cos x .8 and sin x .6, then tanx Find sin s. See Example 1. 5. cos s 3 , s in quadrant I 4 5 , tan s 0 5

6. cot s 8. tan s 10. csc s

1 , s in quadrant IV 3 7 , sec s 0 2 8 5

7. coss 9. sec s

11 , tan s 0 4

9 11.

Why is it unnecessary to give the quadrant of s in Exercise 10?

LIALMC07_0321227638.QXP

2/26/04

10:47 AM

Page 610

610 CHAPTER 7 Trigonometric Identities and Equations 12. 15. 18. 19. 20. sin x 13. odd 14. cos x even 16. tan x 17. odd f x f x f x f x f x f x 5 ; 3

Relating Concepts
For individual or collaborative investigation (Exercises 12 17)

21. cos tan

25 5 ; cot ; 5 2 35 3 ; csc 5 2 26 ; tan 26; 5

A function is called an even function if f x f x for all x in the domain of f. A function is called an odd function if f x f x for all x in the domain of f. Work Exercises 1217 in order, to see the connection between the negative-angle identities and even and odd functions. 12. Complete the statement: sinx 14. Complete the statement: cosx 16. Complete the statement: tanx . . . 13. Is the function dened by f x sin x even or odd? 15. Is the function dened by f x cos x even or odd? 17. Is the function dened by f x tan x even or odd?

sec

22. sin cot csc

6 ; sec 5; 12 56 12 17 ; 17

23. sin cos sec

417 ; cot 4; 17

Concept Check is true. 18.

For each graph, determine whether f x f x or f x f x


4

17 ; csc 17 4 2 24. sin ; 5

19.

221 21 cos ; tan ; 5 21 521 21 ; sec 2 21 3 4 25. sin ; cos ; 5 5 3 5 5 tan ; sec ; csc 4 4 3 3 4 26. cos ; tan ; 5 3 3 5 cot ; sec ; 4 3 5 csc 4 3 7 27. sin ; cos ; 4 4 cot 37 7 tan ; cot ; 3 7 csc 47 7
4 4

20.

Find the remaining ve trigonometric functions of . See Example 1. 21. sin 2 , in quadrant II 3 1 , in quadrant IV 4 22. cos 1 , in quadrant I 5 5 , in quadrant III 2 4 , cos 0 5 1 , sin 0 4

23. tan 25. cot 27. sec

24. csc 26. sin 28. cos

4 , sin 0 3 4 , sin 0 3

LIALMC07_0321227638.QXP

2/26/04

10:47 AM

Page 611

7.1 Fundamental Identities

611

28. sin

15 ; 4 15 ; 15

Concept Check For each expression in Column I, choose the expression from Column II that completes an identity. I cos x 29. sin x 30. tan x 31. cosx 32. tan2 x 1 33. 1 II A. sin2 x cos2 x B. cot x C. sec2 x D. sin x cos x

tan 15; cot

415 sec 4; csc 15 29. B 30. D 31. E 32. C 33. A 34. C 35. A 36. E 37. D 38. B 2x 1 41. sin x1 22p 4 42. tan p 43. sin x 1 cos2 x 44. cot x 1 sin2 x sin x

E. cos x

Concept Check For each expression in Column I, choose the expression from Column II that completes an identity. You may have to rewrite one or both expressions. I 34. tan x cos x 35. sec2 x 1 36. sec x csc x
2

II A. B. sin x cos2 x 1 sec2 x


2

45. tan x sec2 x 1 46. cot x csc2 x 1 1 cos2 x 47. csc x 1 cos2 x 48. 49. 52. 55. 56. 58. 60. 62. 1 sin2 x sec x 1 sin2 x cos 50. 1 51. cot csc2 53. cos2 54. tan2 sec cos 1 cot 57. cot tan sin2 sin2 cos2 59. cos cot tan 61. cos2 tan2

C. sinx D. csc2 x cot 2 x sin2 x E. tan x

37. 1 sin2 x 38. cos x

9 39. 9 40.

A student writes 1 cot 2 csc2. Comment on this students work. Another student makes the following claim: Since sin2 cos2 1, I should be able to also say that sin cos 1 if I take the square root of both sides. Comment on this students statement.
x Suppose that cos x 1 . Find sin . p 4 p .

41. Concept Check 42. Concept Check

Find tan if sec

Write the rst trigonometric function in terms of the second trigonometric function. See Example 2. 43. sin x ; cos x 46. cot x ; csc x 44. cot x ; sin x 47. csc x ; cos x 45. tan x ; sec x 48. sec x ; sin x

Write each expression in terms of sine and cosine, and simplify it. See Example 3. 49. cot sin 52. cot 1 tan
2 2

50. sec cot sin 53. sin csc 1


2 2

51. cos csc 54. sec 1 sec 1

55. 1 cos 1 sec 57. cos2 sin2 sin cos

cos sin 56. sin 58. 1 sin2 1 cot 2 1 tan2 1 cot 2

59. sec cos 61. sin csc sin

60. sec csc cos sin 62.

LIALMC07_0321227638.QXP

2/26/04

10:47 AM

Page 612

612 CHAPTER 7 Trigonometric Identities and Equations 63. sec2 64. sin 256 60 256 60 65. ; 12 12 22 8 22 8 ; 9 9 67. sin2x 68. It is the negative of sin2x. 69. cos4x 70. It is the same function. 71. (a) sin4x (b) cos2x (c) 5 sin3x 72. identity 73. not an identity 74. not an identity 75. identity 76. not an identity 66.

63. sin2 tan2 cos2 65. Concept Check 66. Concept Check

64.

sec x tan x Let cos x 1 . 5 . Find all possible values for sin x cos x Let csc x 3. Find all possible values for sin xsec . x

tan sec

Relating Concepts
For individual or collaborative investigation (Exercises 67 71)

In Chapter 6 we graphed functions dened by y c a f bx d with the assumption that b 0. To see what happens when b 0, work Exercises 67 71 in order. 67. Use a negative-angle identity to write y sin2x as a function of 2x. 68. How does your answer to Exercise 67 relate to y sin2x? 69. Use a negative-angle identity to write y cos4x as a function of 4x. 70. How does your answer to Exercise 69 relate to y cos4x? 71. Use your results from Exercises 6770 to rewrite the following with a positive value of b. (a) sin4x (b) cos2x (c) 5 sin3x

Use a graphing calculator to decide whether each equation is an identity. (Hint: In Exercise 76, graph the function of x for a few different values of y (in radians).) 72. cos 2x 1 2 sin2 x 74. sin x 1 cos x
2

73. 2 sin s sin 2s 75. cos 2x cos2 x sin2 x

76. cosx y cos x cos y

7.2 Verifying Trigonometric Identities


Verifying Identities by Working with One Side
s

Verifying Identities by Working with Both Sides

Recall that an identity is an equation that is satised for all meaningful replacements of the variable. One of the skills required for more advanced work in mathematics, especially in calculus, is the ability to use identities to write expressions in alternative forms. We develop this skill by using the fundamental identities to verify that a trigonometric equation is an identity (for those values of the variable for which it is dened). Here are some hints to help you get started.

LIALMC07_0321227638.QXP

2/26/04

10:47 AM

Page 613

7.2 Verifying Trigonometric Identities

613

Looking Ahead to Calculus


Trigonometric identities are used in calculus to simplify trigonometric expressions, determine derivatives of trigonometric functions, and change the form of some integrals.

Hints for Verifying Identities


1. Learn the fundamental identities given in the last section. Whenever you see either side of a fundamental identity, the other side should come to mind. Also, be aware of equivalent forms of the fundamental identities. For example, sin2 1 cos2 is an alternative form of the identity sin2 cos2 1. 2. Try to rewrite the more complicated side of the equation so that it is identical to the simpler side. 3. It is sometimes helpful to express all trigonometric functions in the equation in terms of sine and cosine and then simplify the result. 4. Usually, any factoring or indicated algebraic operations should be performed. For example, the expression sin2 x 2 sin x 1 can be factored as sin x 12. The sum or difference of two trigonometric expressions, 1 1 such as sin cos , can be added or subtracted in the same way as any other rational expression. 1 1 cos sin sin cos sin cos sin cos cos sin sin cos

TEACHING TIP There is no substitute for experience when it comes to verifying identities. Guide students through several examples, giving hints such as Apply a reciprocal identity, or Use a different form of the Pythagorean identity sin2 cos2 1.

5. As you select substitutions, keep in mind the side you are not changing, because it represents your goal. For example, to verify the identity tan2 x 1 1 , cos2 x

try to think of an identity that relates tan x to cos x. In this case, since 1 2 2 sec x cos x and sec x tan x 1, the secant function is the best link between the two sides. 6. If an expression contains 1 sin x, multiplying both numerator and denominator by 1 sin x would give 1 sin2 x, which could be replaced with cos2 x. Similar results for 1 sin x, 1 cos x, and 1 cos x may be useful.

CAUTION Verifying identities is not the same as solving equations. Techniques used in solving equations, such as adding the same terms to both sides, or multiplying both sides by the same term, should not be used when working with identities since you are starting with a statement (to be veried) that may not be true.

Verifying Identities by Working with One Side To avoid the temptation to use algebraic properties of equations to verify identities, work with only one side and rewrite it to match the other side, as shown in Examples 1 4.

LIALMC07_0321227638.QXP

2/26/04

10:47 AM

Page 614

614 CHAPTER 7 Trigonometric Identities and Equations

EXAMPLE 1 Verifying an Identity (Working with One Side)

Verify that the following equation is an identity. cot s 1 csc s cos s sin s
Solution

We use the fundamental identities from Section 7.1 to rewrite one side of the equation so that it is identical to the other side. Since the right side is more complicated, we work with it, using the third hint to change all functions to sine or cosine.

Steps
For s = x, cot x + 1 = csc x (cos x + sin x)
4

Reasons

Right side of given equation

csc s cos s sin s


2

1 1 cos s sin s csc s sin s sin s cos s sin s sin s sin s



Distributive property (Section R.1)
cos s sin s s cot s ; sin sin s 1

cot s 1
4 The graphs coincide, supporting the conclusion in Example 1.

Left side of given equation

The given equation is an identity since the right side equals the left side.
Now try Exercise 33.

EXAMPLE 2 Verifying an Identity (Working with One Side)

Verify that the following equation is an identity. tan2 x 1 cot 2 x


Solution

1 1 sin2 x

We work with the more complicated left side, as suggested in the second hint. Again, we use the fundamental identities from Section 7.1. tan2 x 1 cot 2 x tan2 x tan2 x cot 2 x
Distributive property
1 cot 2 x tan 2 x 1 tan2 x tan 2 x 1

tan2 x

(1 +

cot2 x)

1 = 2x 1 sin 4

tan2 x tan2 x tan2 x 1 sec2 x


2

1 tan2 x

tan2 x 1 sec2 x
1 sec2 x cos 2 x

1 cos2 x 1 1 sin2 x

4 The screen supports the conclusion in Example 2.

cos2 x 1 sin2 x

Since the left side is identical to the right side, the given equation is an identity.
Now try Exercise 37.

LIALMC07_0321227638.QXP

2/26/04

10:47 AM

Page 615

7.2 Verifying Trigonometric Identities

615

EXAMPLE 3 Verifying an Identity (Working with One Side)

Verify that the following equation is an identity. tan t cot t sec2 t csc2 t sin t cos t
Solution

We transform the more complicated left side to match the right side.
ab c b a c c (Section R.5)

tan t cot t tan t cot t sin t cos t sin t cos t sin t cos t tan t 1 1 cot t sin t cos t sin t cos t

a b

a1 b

sin t 1 cos t 1 cos t sin t cos t sin t sin t cos t 1 1 2 cos t sin2 t

sin t cos t tan t cos t ; cot t sin t

sec2 t csc2 t

1 cos2 t

1 2 sec2 t ; sin 2 t csc t

The third hint about writing all trigonometric functions in terms of sine and cosine was used in the third line of the solution.
Now try Exercise 41.

EXAMPLE 4 Verifying an Identity (Working with One Side)


TEACHING TIP Show that the identity in Example 4 can also be veried by multiplying the numerator and denominator of the left side by 1 sin x.

Verify that the following equation is an identity. cos x 1 sin x 1 sin x cos x
Solution

We work on the right side, using the last hint in the list given earlier to multiply numerator and denominator on the right by 1 sin x. 1 sin x 1 sin x 1 sin x Multiply by 1. (Section R.1) cos x cos x 1 sin x 1 sin2 x cos x 1 sin x cos2 x cos x 1 sin x cos x 1 sin x
x y x y x 2 y 2 (Section R.3) 1 sin2 x cos2 x Lowest terms (Section R.5)

Now try Exercise 47.

Verifying Identities by Working with Both Sides If both sides of an identity appear to be equally complex, the identity can be veried by working independently on the left side and on the right side, until each side is changed into some common third result. Each step, on each side, must be reversible.

LIALMC07_0321227638.QXP

2/26/04

10:47 AM

Page 616

616 CHAPTER 7 Trigonometric Identities and Equations

left

right

common third expression

With all steps reversible, the procedure is as shown in the margin. The left side leads to a common third expression, which leads back to the right side. This procedure is just a shortcut for the procedure used in Examples 1 4: one side is changed into the other side, but by going through an intermediate step.
EXAMPLE 5 Verifying an Identity (Working with Both Sides)

Verify that the following equation is an identity. sec tan 1 2 sin sin2 sec tan cos2
Solution

Both sides appear equally complex, so we verify the identity by changing each side into a common third expression. We work rst on the left, multiplying numerator and denominator by cos . sec tan sec tan cos sec tan sec tan cos sec cos tan cos sec cos tan cos 1 tan cos 1 tan cos 1
Multiply by 1.

Distributive property sec cos 1

sin cos cos sin 1 cos cos 1 sin 1 sin

sin tan cos

On the right side of the original equation, begin by factoring. 1 2 sin sin2 1 sin 2 cos2 cos2 We have shown that
Left side of given equation Common third expression Right side of given equation x 2 2xy y 2 x y2
(Section R.4)

1 sin 1 sin2

cos2 1 sin2

1 sin 2 Factor 1 sin2 . 1 sin 1 sin 1 sin 1 sin


Lowest terms

sec tan 1 sin 1 2 sin sin2 , sec tan 1 sin cos2

verifying that the given equation is an identity.


Now try Exercise 51.

LIALMC07_0321227638.QXP

2/26/04

10:47 AM

Page 617

7.2 Verifying Trigonometric Identities

617

CAUTION

Use the method of Example 5 only if the steps are reversible.

There are usually several ways to verify a given identity. For instance, another way to begin verifying the identity in Example 5 is to work on the left as follows. sin 1 sec tan cos cos sec tan 1 sin cos cos 1 sin cos 1 sin cos 1 sin 1 sin

Fundamental identities (Section 7.1)

Add and subtract fractions.


(Section R.5)

Simplify the complex fraction.


(Section R.5)

Compare this with the result shown in Example 5 for the right side to see that the two sides indeed agree.
EXAMPLE 6 Applying a Pythagorean Identity to Radios

Tuners in radios select a radio station by adjusting the frequency. A tuner may contain an inductor L and a capacitor C, as illustrated in Figure 3. The energy stored in the inductor at time t is given by Lt k sin22Ft and the energy stored in the capacitor is given by Ct k cos22Ft, where F is the frequency of the radio station and k is a constant. The total energy E in the circuit is given by
L C

Et Lt Ct. Show that E is a constant function. (Source: Weidner, R. and R. Sells, Elementary Classical Physics, Vol. 2, Allyn & Bacon, 1973.)
Solution

An Inductor and a Capacitor


Figure 3

Et Lt Ct k sin 2Ft k cos 2Ft


2 2

Given equation Substitute. Factor. (Section R.4) sin2 cos2 1 (Here 2Ft).
2

k sin 2Ft cos 2Ft


2

k1 k

Since k is constant, Et is a constant function.


Now try Exercise 85.

LIALMC07_0321227638.QXP

2/26/04

10:47 AM

Page 618

618 CHAPTER 7 Trigonometric Identities and Equations

7.2 Exercises
1 sin cos 1 2. csc x sec x or sin x cos x 3. 1 sec s 4. 1 cot 2 5. 1 6. 2 or 2 sec2 cos 1 7. 1 8. or csc sin 9. 2 2 sin t 10. sec2 s 2 cos x 11. or 2 cot x csc x sin2 x 12. 1 2 sin cos 13. sin 1 sin 1 14. sec 1 sec 1 15. 4 sin x 16. 4 tan x cot x or 4 17. 2 sin x 1 sin x 1 18. 4 tan 3 tan 1 19. cos2 x 12 20. cot 2 x 2 cot 2 x 1 or csc2 x cot 2 x 2 21. sin x cos x 1 sin x cos x 22. sin cos 1 sin cos 23. sin 24. cos 25. 1 26. 1 27. tan2 28. sec2 29. tan2 x 30. cot 2 t 31. sec2 x 32. csc2 1. csc sec or

Perform each indicated operation and simplify the result. 1. cot 1 cot 2. 5. 8. 11. sec x csc x csc x sec x 1 1 csc2 sec2 cos sin sin 1 cos 1 1 1 cos x 1 cos x 3. tan s cot s csc s 6. 1 1 sin 1 sin 1

4. cos sec csc 7. cos x sin x sec x csc x

9. 1 sin t2 cos2 t 12. sin cos 2

10. 1 tan s2 2 tan s

Factor each trigonometric expression. 13. sin2 1 15. sin x 12 sin x 12 17. 2 sin2 x 3 sin x 1 19. cos x 2 cos x 1
4 2

14. sec2 1 16. tan x cot x2 tan x cot x2 18. 4 tan2 tan 3 20. cot 4 x 3 cot 2 x 2 22. sin3 cos3

21. sin3 x cos3 x

Each expression simplies to a constant, a single function, or a power of a function. Use fundamental identities to simplify each expression. 23. tan cos 26. cot t tan t 29. sec2 x 1 32. 1 cot tan tan2 24. cot sin 27. sin tan cos 25. sec r cos r 28. 31. csc sec cot sin2 x sin x csc x cos2 x

30. csc2 t 1

In Exercises 33 68, verify that each trigonometric equation is an identity. See Examples 1 5. 33. 35. cot cos csc 1 sin2 cos cos 34. 36. tan sin sec tan2 1 sec sec

37. cos2 tan2 1 1 39. cot s tan s sec s csc s 41. cos sin sec2 tan2 sec csc

38. sin2 1 cot 2 1 40. sin2 tan2 cos2 sec2 42. 44. sin2 sec cos cos cos 1 sin cot

43. sin4 cos4 2 sin2 1 46. tan2 sin2 tan2 cos2 1

45. 1 cos2 1 cos2 2 sin2 sin4

LIALMC07_0321227638.QXP

2/26/04

10:47 AM

Page 619

7.2 Verifying Trigonometric Identities

619

47. 49. 50. 51. 52. 53. 54.

cos 1 cos tan2 sec 1 1 1 2 sec2 1 sin 1 sin 1 sec tan sec tan

48.

sec tan 2 1 2 tan sec csc tan csc

tan s sin s cot s sec s csc s 1 cos s 1 cos s 1 cos x cot x csc x2 1 cos x cot 1 1 tan cot 1 1 tan 1 1 2 tan tan sec tan sec csc cot cot csc tan sin 1 sin sec2 2 sec tan tan2 1 sin sin cos cos sin 1 1 sin cos

55. sin2 sec2 sin2 csc2 sec2 56.

57. sec4 x sec2 x tan4 x tan2 x 58.

59. sin cos

60. 61. 62. 63.

sin sin cos csc 1 cos2 1 cos 1 cos sec4 s tan4 s sec2 s tan2 s sec2 s tan2 s cot 2 t 1 1 2 sin2 t 1 cot 2 t tan2 t 1 tan t cot t sec2 t tan t cot t 1 cos x 1 cos x 4 cot x csc x 1 cos x 1 cos x 1 sin 1 sin

64. 1 sin x cos x2 21 sin x 1 cos x 65.

66. sec tan 2

67. sec csc cos sin cot tan 68. sin4 cos4 1 sin2 cos2 A student claims that the equation cos sin 1 is an identity, since by letting 90 or 2 radians we get 0 1 1, a true statement. Comment on this students reasoning. An equation that is an identity has an innite number of solutions. If an equation has an innite number of solutions, is it necessarily an identity? Explain.

9 69. 9 70.

LIALMC07_0321227638.QXP

2/26/04

10:47 AM

Page 620

620 CHAPTER 7 Trigonometric Identities and Equations 71. sec tan 1 sin cos 72. csc cot sec 1 tan cos 1 73. cot sin tan 74. tan sin cos sec 75. identity 76. identity 77. not an identity 78. not an identity 83. It is true when sin x 0. 84. (a) I k1 sin2 (b) For 2 n for all integers n, cos2 1, its maximum value, and I attains a maximum value of k. 85. (a) P 16k cos22 t (b) P 16k 1 sin22 t

Concept Check Graph each expression and conjecture an identity. Then verify your conjecture algebraically. 71. sec tan 1 sin 73. cos 1 sin tan 72. csc cot sec 1 74. tan sin cos

Graph the expressions on each side of the equals sign to determine whether the equation might be an identity. (Note: Use a domain whose length is at least 2.) If the equation looks like an identity, prove it algebraically. See Example 1. 75. 77. 2 5 cos s 2 csc s 5 cot s sin s tan s cot s 2 sin2 s tan s cot s 76. 1 cot 2 s 78. sec2 s sec2 s 1

1 1 sec2 s 1 sin s 1 sin s

By substituting a number for s or t, show that the equation is not an identity. 79. sincsc s 1 81. csc t 1 cot t
2

80. cos2 s cos s 82. cos t 1 sin2 t

83. When is sin x 1 cos2 x a true statement? (Modeling) Work each problem. 84. Intensity of a Lamp According to Lamberts law, the intensity of light from a single source on a at surface at point P is given by I k cos2 , where k is a constant. (Source: Winter, C., Solar Power Plants, Springer-Verlag, 1991.)

9(b) Explain

(a) Write I in terms of the sine function. why the maximum value of I occurs when 0.
y

85. Oscillating Spring The distance or displacement y of a weight attached to an oscillating spring from its natural position is modeled by y 4 cos2 t, where t is time in seconds. Potential energy is the energy of position and is given by P ky 2,

where k is a constant. The weight has the greatest potential energy when the spring is stretched the most. (Source: Weidner, R. and R. Sells, Elementary Classical Physics, Vol. 1, Allyn & Bacon, 1973.) (a) Write an expression for P that involves the cosine function. (b) Use a fundamental identity to write P in terms of sin2 t. 86. Radio Tuners Refer to Example 6. Let the energy stored in the inductor be given by Lt 3 cos26,000,000t and the energy in the capacitor be given by Ct 3 sin26,000,000t,

LIALMC07_0321227638.QXP

2/26/04

10:47 AM

Page 621

7.3 Sum and Difference Identities

621

86. (a) The sum of L and C equals 3.


4

where t is time in seconds. The total energy E in the circuit is given by Et Lt Ct. (a) Graph L, C, and E in the window 0, 10 6 by 1, 4, with Xscl 10 7 and Yscl 1. Interpret the graph. (b) Make a table of values for L, C, and E starting at t 0, incrementing by 10 7. Interpret your results. (c) Use a fundamental identity to derive a simplied expression for Et.
(b) Let Y1 Lt, Y2 Ct, and Y3 Et. Y3 3 for all inputs. (c) Et 3

0 1

10 6

7.3 Sum and Difference Identities


Cosine Sum and Difference Identities Difference Identities
s

Cofunction Identities

Sine and Tangent Sum and

Cosine Sum and Difference Identities Several examples presented earlier should have convinced you by now that cosA B does not equal cos A cos B. For example, if A 2 and B 0, then
0 cos 0, 2 2 cos 0 0 1 1. cos A cos B cos 2
cosA B cos

while

We can now derive a formula for cosA B. We start by locating angles A and B in standard position on a unit circle, with B A. Let S and Q be the points where the terminal sides of angles A and B, respectively, intersect the circle. Locate point R on the unit circle so that angle POR equals the difference A B. See Figure 4.
y

(cos(A B), sin(A B)) R Q (cos B, sin B (cos A, sin A) S A A B P (1, 0) x B

Figure 4

Point Q is on the unit circle, so by the work with circular functions in Chapter 6, the x-coordinate of Q is the cosine of angle B, while the y-coordinate of Q is the sine of angle B. Q has coordinates cos B, sin B. In the same way, S has coordinates cos A, sin A, and R has coordinates cosA B, sinA B.

LIALMC07_0321227638.QXP

2/26/04

10:47 AM

Page 622

622 CHAPTER 7 Trigonometric Identities and Equations

Angle SOQ also equals A B. Since the central angles SOQ and POR are equal, chords PR and SQ are equal. By the distance formula, since PR SQ, cosA B 1 2 sinA B 0 2 cos A cos B2 sin A sin B2. Squaring both sides and clearing parentheses gives cos2A B 2 cosA B 1 sin2A B cos2 A 2 cos A cos B cos2 B sin2 A 2 sin A sin B sin2 B. Since sin2 x cos2 x 1 for any value of x, we can rewrite the equation as 2 2 cosA B 2 2 cos A cos B 2 sin A sin B cosA B cos A cos B sin A sin B. Subtract 2; divide by 2. Although Figure 4 shows angles A and B in the second and rst quadrants, respectively, this result is the same for any values of these angles. To nd a similar expression for cosA B, rewrite A B as A B and use the identity for cosA B. cosA B cosA B cos A cosB sin A sinB Cosine difference identity cos A cos B sin Asin B cosA B cos A cos B sin A sin B
Negative-angle identities
(Section 7.1)

(Section 2.1)

Cosine of a Sum or Difference


cos(A B) cos A cos B sin A sin B cos(A B) cos A cos B sin A sin B

These identities are important in calculus and useful in certain applications. Although a calculator can be used to nd an approximation for cos 15, for example, the method shown below can be applied to get an exact value, as well as to practice using the sum and difference identities.
EXAMPLE 1 Finding Exact Cosine Function Values

Find the exact value of each expression. (a) cos 15


Solution

(b) cos

5 12

(c) cos 87 cos 93 sin 87 sin 93

(a) To nd cos 15, we write 15 as the sum or difference of two angles with known function values. Since we know the exact trigonometric function values of 45 and 30, we write 15 as 45 30. (We could also use 60 45.) Then we use the identity for the cosine of the difference of two angles.

LIALMC07_0321227638.QXP

2/26/04

10:47 AM

Page 623

7.3 Sum and Difference Identities

623

TEACHING TIP In Example 1(b),


students may benet from con5 verting radians to 75 in 12 order to realize that

cos 15 cos45 30 cos 45 cos 30 sin 45 sin 30 Cosine difference identity (b) cos 2 3 2 1 2 2 2 2 6 2 4
(Section 5.3)

30 45 6 4
can be used in place of 5 . 12

5 cos 12 6 4 cos

The screen supports the solution in Example 1(b) by showing that cos 5 = 6 2 . 4 12

cos sin sin 6 4 6 4 1 2 3 2 2 2 2 2 6 2 4

3 2 12 ; 4 12

Cosine sum identity


(Section 6.2)

(c) cos 87 cos 93 sin 87 sin 93 cos87 93


Cosine sum identity

cos 180 1
(Section 5.2)

Now try Exercises 7, 9, and 11.

Cofunction Identities We can use the identity for the cosine of the difference of two angles and the fundamental identities to derive cofunction identities.

TEACHING TIP Mention that these


identities state that the trigonometric function of an acute angle is the same as the cofunction of its complement. Verify the cofunction identities for acute angles using complementary angles in a right triangle along with the righttriangle-based denitions of the trigonometric functions. Emphasize that these identities apply to any angle , not just acute angles.

Cofunction Identities
cos(90 ) sin sin(90 ) cos tan(90 ) cot cot(90 ) tan sec(90 ) csc csc(90 ) sec

Similar identities can be obtained for a real number domain by replacing 90 with 2 .

Substituting 90 for A and for B in the identity for cosA B gives cos90 cos 90 cos sin 90 sin 0 cos 1 sin sin . This result is true for any value of since the identity for cosA B is true for any values of A and B.

LIALMC07_0321227638.QXP

2/26/04

10:47 AM

Page 624

624 CHAPTER 7 Trigonometric Identities and Equations

EXAMPLE 2 Using Cofunction Identities to Find


TEACHING TIP Verify results from
Example 2 using a calculator.

Find an angle that satises each of the following. (a) cot tan 25
Solution

(b) sin cos30

(c) csc

3 sec 4

(a) Since tangent and cotangent are cofunctions, tan90 cot . cot tan 25 tan90 tan 25 Cofunction identity 90 25
Set angle measures equal.

65
(b) sin cos30 cos90 cos30 Cofunction identity 90 30

120
(c)

sec

3 sec Cofunction identity 2 4 4


sec
Combine terms.

sec

csc

3 sec 4

4
Now try Exercises 25 and 27.

Because trigonometric (circular) functions are periodic, the solutions in Example 2 are not unique. We give only one of innitely many possibilities.
NOTE

If one of the angles A or B in the identities for cosA B and cosA B is a quadrantal angle, then the identity allows us to write the expression in terms of a single function of A or B.
EXAMPLE 3 Reducing cos A B to a Function of a Single Variable

Write cos180 as a trigonometric function of .


Solution

Use the difference identity. Replace A with 180 and B with . 1 cos 0 sin cos
Now try Exercise 39.
(Section 5.2)

cos180 cos 180 cos sin 180 sin

LIALMC07_0321227638.QXP

2/26/04

10:47 AM

Page 625

7.3 Sum and Difference Identities

625

Sine and Tangent Sum and Difference Identities We can use the cosine sum and difference identities to derive similar identities for sine and tangent. Since sin cos90 , we replace with A B to get
sinA B cos90 A B cos90 A B cos90 A cos B sin90 A sin B
Cosine difference identity Cofunction identity

sinA B sin A cos B cos A sin B. Cofunction identities Now we write sinA B as sinA B and use the identity for sinA B. sinA B sinA B sin A cosB cos A sinB Sine sum identity sinA B sin A cos B cos A sin B
Negative-angle identities

Sine of a Sum or Difference


sin(A B) sin A cos B cos A sin B sin(A B) sin A cos B cos A sin B To derive the identity for tanA B, we start with tanA B sinA B cosA B sin A cos B cos A sin B . cos A cos B sin A sin B
Fundamental identity
(Section 7.1)

Sum identities

We express this result in terms of the tangent function by multiplying both nu1 merator and denominator by cos A cos B . 1 sin A cos B cos A sin B 1 cos A cos B tanA B cos A cos B sin A sin B 1 1 cos A cos B cos A sin B sin A cos B cos A cos B cos A cos B cos A cos B sin A sin B cos A cos B cos A cos B sin B sin A cos A cos B sin A sin B 1 cos A cos B tanA B tan A tan B 1 tan A tan B
sin tan cos

Simplify the complex fraction.


(Section R.5)

Multiply numerators; multiply denominators.

Replacing B with B and using the fact that tanB tan B gives the identity for the tangent of the difference of two angles.

LIALMC07_0321227638.QXP

2/26/04

10:47 AM

Page 626

626 CHAPTER 7 Trigonometric Identities and Equations

Tangent of a Sum or Difference


tan(A B) tan A tan B 1 tan A tan B tan(A B) tan A tan B 1 tan A tan B

EXAMPLE 4 Finding Exact Sine and Tangent Function Values

Find the exact value of each expression. (a) sin 75


Solution

(b) tan

7 12

(c) sin 40 cos 160 cos 40 sin 160

(a) sin 75 sin45 30 sin 45 cos 30 cos 45 sin 30 Sine sum identity (b) tan 2 3 2 1 2 2 2 2 6 2 4
(Section 5.3)

7 tan 12 3 4

tan 3 tan 4 1 tan 3 tan 4

Tangent sum identity


(Section 6.2)

3 1 1 3 1 3 1 1 3 1 3 1 3

Rationalize the denominator. (Section R.7)

3 3 1 3 Multiply. (Section R.7) 13


Combine terms. Factor out 2. (Section R.5) Lowest terms Sine difference identity

4 23 2 2 2 3 2 2 3

(c) sin 40 cos 160 cos 40 sin 160 sin40 160 sin120 sin 120 3 2
Negative-angle identity
(Section 5.3)

Now try Exercises 29, 31, and 35.

LIALMC07_0321227638.QXP

2/26/04

10:47 AM

Page 627

7.3 Sum and Difference Identities

627

EXAMPLE 5 Writing Functions as Expressions Involving Functions of

Write each function as an expression involving functions of . (a) sin30


Solution

(b) tan45

(c) sin180

(a) Using the identity for sinA B, sin30 sin 30 cos cos 30 sin (b) tan45 1 3 cos sin . 2 2

tan 45 tan 1 tan 1 tan 45 tan 1 tan

(c) sin180 sin 180 cos cos 180 sin 0 cos 1 sin sin
Now try Exercises 43 and 47.

EXAMPLE 6 Finding Function Values and the Quadrant of A B

Suppose that A and B are angles in standard position, with sin A 4 5, 5 3 2 A , and cos B 13 , B 2 . Find each of the following. (a) sinA B
Solution

(b) tanA B

(c) the quadrant of A B

(a) The identity for sinA B requires sin A, cos A, sin B, and cos B. We are given values of sin A and cos B. We must nd values of cos A and sin B. sin2 A cos2 A 1 16 cos2 A 1 25 cos2 A 9 25 3 5
Fundamental identity sin A 4 5 Subtract 16 25 . Since A is in quadrant II, cos A 0.

cos A

In the same way, sin B 12 13 . Now use the formula for sinA B. sinA B 4 5


5 3 13 5 12 13 4 3

20 36 16 65 65 65

(b) To nd tanA B, rst use the values of sine and cosine from part (a) to 4 12 get tan A 3 and tan B 5 . tanA B 1
4 12 3 5


12 5

16 15

48 15

63
15

16 15

16 63

LIALMC07_0321227638.QXP

2/26/04

10:47 AM

Page 628

628 CHAPTER 7 Trigonometric Identities and Equations

(c) From parts (a) and (b), sinA B 65 and tanA B 63 , both positive. Therefore, A B must be in quadrant I, since it is the only quadrant in which both sine and tangent are positive.
Now try Exercise 51.

16

16

EXAMPLE 7 Applying the Cosine Difference Identity to Voltage

Common household electric current is called alternating current because the current alternates direction within the wires. The voltage V in a typical 115-volt outlet can be expressed by the function Vt 163 sin t, where is the angular speed (in radians per second) of the rotating generator at the electrical plant and t is time measured in seconds. (Source: Bell, D., Fundamentals of Electric Circuits, Fourth Edition, Prentice-Hall, 1988.) (a) It is essential for electric generators to rotate at precisely 60 cycles per sec so household appliances and computers will function properly. Determine for these electric generators. (b) Graph V in the window 0, .05 by 200, 200. (c) Determine a value of so that the graph of Vt 163 cost is the same as the graph of Vt 163 sin t.
Solution

(a) Each cycle is 2 radians at 60 cycles per sec, so the angular speed is 602 120 radians per sec. (b) Vt 163 sin t 163 sin 120 t. Because the amplitude of the function is 163 (from Section 6.3), 200, 200 is an appropriate interval for the range, as shown in Figure 5.
For x = t, V( t ) = 163 sin 120 t
200

.05

200
Figure 5

(c) Using the negative-angle identity for cosine and a cofunction identity, cos x

Therefore, if 2 , then


2
cos

x 2

cos

x sin x. 2

Vt 163 cos t

163 sin t.
Now try Exercise 81.

LIALMC07_0321227638.QXP

2/26/04

10:47 AM

Page 629

7.3 Sum and Difference Identities

629

7.3 Exercises
1. F 2. A 3. C 4. D 6 2 6 2 5. 6. 4 4 2 6 7. 4 2 6 8. 4

Concept Check Match each expression in Column I with the correct expression in Column II to form an identity. I 1. cosx y 2. cosx y 3. sinx y 4. sinx y II A. cos x cos y sin x sin y B. sin x sin y cos x cos y C. sin x cos y cos x sin y D. sin x cos y cos x sin y E. cos x sin y sin x cos y F. cos x cos y sin x sin y Use identities to nd each exact value. (Do not use a calculator.) See Example 1. cos 75 cos 105 (Hint: 105 60 45) cos 7 12 6. cos15 8. cos105 (Hint: 105 60 45) 10. cos 12. cos

6 2 2 6 9. 10. 4 4 11. 0 12. 1 13. cot 3 5 14. cos 75 15. sin 12 16. cos 17. cos 10 8 18. tan 20. 22. 25. 28. 30. 32.

5. 2 19. csc56 42 7. 5 cot84 3 21. tan cos 23. cos 24. tan 9. 100 15 26. 27. 20 3 80 6 2 11. 29. 3 4 2 3 31. 2 3
33. 35. 6 2 4


12 12

cos 40 cos 50 sin 40 sin 50

7 2 7 2 cos sin sin 9 9 9 9

6 2 4

Write each function value in terms of the cofunction of a complementary angle. See Example 2. 13. tan 87 14. sin 15 18. cot 9 10 15. cos 16. sin 2 5

34. 2 3 37. 1

2 36. 1 2

38. 1

17. sin

5 8

19. sec 146 42

20. tan 174 3

Use the cofunction identities to ll in each blank with the appropriate trigonometric function name. See Example 2. 21. cot 23.

3 6 33 sin 57

22. sin 24.

2 3

72 cot 18

Find an angle that makes each statement true. See Example 2. 25. tan cot45 2 27. sin3 15 cos 25 26. sin cos2 10 28. cot 10 tan2 20

Use identities to nd the exact value of each of the following. See Example 4. 29. sin 32. sin 5 12 30. tan 5 12 31. tan

7 7 33. sin 34. tan 12 12 12 35. sin 76 cos 31 cos 76 sin 31 36. sin 40 cos 50 cos 40 sin 50 tan 80 tan55 tan 80 tan 55 37. 38. 1 tan 80 tan 55 1 tan 80 tan55

12

LIALMC07_0321227638.QXP

2/26/04

10:47 AM

Page 630

630 CHAPTER 7 Trigonometric Identities and Equations 39. sin 40. cos 41. sin x 42. sin x 2 cos sin 43. 2 44. sin 1 45. cos x 3 sin x 2 2 cos x sin x 46. 2 3 tan 47. 1 3 tan 3 tan 1 48. 3 tan 1 tan x 3 tan x 1 49. 50. 1 tan x 3 tan x 16 33 63 51. (a) (b) (c) (d) I 65 65 16 4 66 86 3 52. (a) (b) 25 25 86 3 (c) (d) IV 4 66 210 2 53. (a) 9 42 5 (b) 9 85 52 (c) (d) II 20 210 16 33 63 54. (a) (b) (c) (d) IV 65 65 16 36 13 77 55. (a) (b) (c) 85 85 36 (d) II 77 84 36 56. (a) (b) (c) (d) III 85 85 77 6 2 58. 4 6 2 59. 4 2 6 60. (a) 4 6 2 (b) 4 6 2 61. 62. 2 3 4 6 2 63. 64. 2 3 4 6 2 65. 2 3 66. 4

Use identities to write each expression as a function of x or . See Examples 3 and 5. 39. cos90 42. cos 45. sin


x 2
5 x 6

40. cos180 43. sin45 46. sin

41. cos x


3 2

44. sin180 47. tan60 50. tan

48. tan 30

49. tan x


x 4 6


x 4

Use the given information to nd (a) coss t, (b) sins t, (c) tans t, and (d) the quadrant of s t. See Example 6. 51. cos s 3 5 and sin t , s and t in quadrant I 5 13 1 3 and sin t , s and t in quadrant II 5 5

52. cos s 53. sin s 54. sin s

2 1 and sin t , s in quadrant II and t in quadrant IV 3 3 3 12 and sin t , s in quadrant I and t in quadrant III 5 13 8 3 and cos t , s and t in quadrant III 17 5 15 4 and sin t , s in quadrant II and t in quadrant I 17 5

55. cos s 56. cos s

Relating Concepts
For individual or collaborative investigation (Exercises 57 60)

The identities for cosA B and cosA B can be used to nd exact values of expressions like cos 195 and cos 255, where the angle is not in the rst quadrant. Work Exercises 57 60 in order, to see how this is done. 57. By writing 195 as 180 15, use the identity for cosA B to express cos 195 as cos 15. 58. Use the identity for cosA B to nd cos 15. 59. By the results of Exercises 57 and 58, cos 195 (a) cos 255 11 12 . 60. Find each exact value using the method shown in Exercises 57 59. (b) cos

Find each exact value. Use the technique developed in Relating Concepts Exercises 57 60. 61. sin 165 64. tan 285 62. tan 165 65. tan 11 12 63. sin 255 66. sin


13 12

LIALMC07_0321227638.QXP

2/26/04

10:47 AM

Page 631

7.3 Sum and Difference Identities

631

71. sin 72.

1 tan x tan x 1 tan x 4

x cos x 2

9 68. 9 69.

67. Use the identity cos90 sin , and replace with 90 A, to derive the identity cos A sin90 A. Explain how the identities for secA B, cscA B, and cotA B can be found by using the sum identities given in this section. Why is it not possible to use a method similar to that of Example 5(c) to nd a formula for tan270 ?

79. 3 80. 163 and 163; no 81. (a) 425 lb (c) 0

70. Concept Check Show that if A, B, and C are angles of a triangle, then sinA B C 0. Graph each expression and use the graph to conjecture an identity. Then verify your conjecture algebraically. 71. sin


x 2

72.

1 tan x 1 tan x

Verify that each equation is an identity. 73. sinx y sinx y 2 sin x cos y 74. tanx y tan y x 75. 76. 77. 78. 2tan x tan y 1 tan x tan y

cos tan cot cos sin sins t tan s tan t cos s cos t sinx y tan x tan y sinx y tan x tan y sins t coss t sin s sin t cos t sin t cos t

Exercises 79 and 80 refer to Example 7. 79. How many times does the current oscillate in .05 sec? 80. What are the maximum and minimum voltages in this outlet? Is the voltage always equal to 115 volts? (Modeling) Solve each problem. 81. Back Stress If a person bends at the waist with a straight back making an angle of degrees with the horizontal, then the force F exerted on the back muscles can be modeled by the equation F .6W sin 90 , sin 12

where W is the weight of the person. (Source: Metcalf, H., Topics in Classical Biophysics, Prentice-Hall, 1980.) (a) Calculate F when W 170 lb and 30. (b) Use an identity to show that F is approximately equal to 2.9W cos . (c) For what value of is F maximum?

LIALMC07_0321227638.QXP

2/26/04

10:47 AM

Page 632

632 CHAPTER 7 Trigonometric Identities and Equations 82. (a) 408 lb (b) 46.1 83. (a) The pressure P is oscillating. For x = t, P( t ) =
.05
.4 cos 10

82. Back Stress

Refer to Exercise 81.

1026t [20 ] 4.9

(a) Suppose a 200-lb person bends at the waist so that 45. Estimate the force exerted on the persons back muscles. (b) Approximate graphically the value of that results in the back muscles exerting a force of 400 lb. 83. Sound Waves Sound is a result of waves applying pressure to a persons eardrum. For a pure sound wave radiating outward in a spherical shape, the trigonometric function dened by P a 2 r ct cos r

.05

.05

(b) The pressure oscillates and amplitude decreases as r increases. For x = r, 2 r P(r) = 3 r cos 4.9 10,260 2

can be used to model the sound pressure at a radius of r feet from the source, where t is time in seconds, is length of the sound wave in feet, c is speed of sound in feet per second, and a is maximum sound pressure at the source measured in pounds per square foot. (Source: Beranek, L., Noise and Vibration Control, Institute of Noise Control Engineering, Washington, D.C., 1988.) Let 4.9 ft and c 1026 ft per sec. (a) Let a .4 lb per ft 2. Graph the sound pressure at distance r 10 ft from its source in the window 0, .05 by .05, .05. Describe P at this distance. (b) Now let a 3 and t 10. Graph the sound pressure in the window 0, 20] by 2, 2. What happens to pressure P as radius r increases? (c) Suppose a person stands at a radius r so that r n, where n is a positive integer. Use the difference identity for cosine to simplify P in this situation. 84. Voltage of a Circuit When the two voltages and V2 40 cos 120 t V1 30 sin 120 t

20

(c) P

a cos ct n

84. (a) For x = t, V = V1 + V2 = 30 sin 120 t + 40 cos 120 t


60

are applied to the same circuit, the resulting voltage V will be equal to their sum. (Source: Bell, D., Fundamentals of Electric Circuits, Second Edition, Reston Publishing Company, 1981.) (a) Graph the sum in the window 0, .05 by 60, 60. (b) Use the graph to estimate values for a and so that V a sin120 t . (c) Use identities to verify that your expression for V is valid.

.05

60

(b) a 50; 5.353

7.4 Double-Angle Identities and Half-Angle Identities


Double-Angle Identities
s

Product-to-Sum and Sum-to-Product Identities

Half-Angle Identities

Double-Angle Identities When A B in the identities for the sum of two angles, these identities are called the double-angle identities. For example, to derive an expression for cos 2 A, we let B A in the identity cosA B cos A cos B sin A sin B.
TEACHING TIP A common error is
to write cos 2 A as 2 cos A.

cos 2 A cosA A cos A cos A sin A sin A Cosine sum identity (Section 7.3) cos 2 A cos2 A sin2 A

LIALMC07_0321227638.QXP

2/26/04

10:47 AM

Page 633

7.4 Double-Angle Identities and Half-Angle Identities

633

Two other useful forms of this identity can be obtained by substituting either cos2 A 1 sin2 A or sin2 A 1 cos2 A. Replace cos2 A with the expression 1 sin2 A to get cos 2 A cos2 A sin2 A 1 sin2 A sin2 A Fundamental identity (Section 7.1) cos 2 A 1 2 sin2 A, or replace sin2 A with 1 cos2 A to get cos 2 A cos2 A sin2 A cos2 A 1 cos2 A Fundamental identity cos2 A 1 cos2 A cos 2 A 2 cos2 A 1. We nd sin 2 A with the identity sinA B sin A cos B cos A sin B, letting B A. sin 2 A sinA A sin A cos A cos A sin A Sine sum identity sin 2 A 2 sin A cos A
TEACHING TIP Students might nd
it helpful to see each formula illustrated with a concrete example that they can check. For instance, you might show that cos 60 cos2 30 cos2 30 sin2 30.

Using the identity for tanA B, we nd tan 2 A. tan 2 A tanA A tan 2 A tan A tan A Tangent sum identity 1 tan A tan A 2 tan A 1 tan2 A

Looking Ahead to Calculus


The identities cos 2 A 1 2 sin2 A and cos 2 A 2 cos A 1
2

Double-Angle Identities
cos 2 A cos2 A sin2 A cos 2 A 2 cos2 A 1 tan 2 A cos 2 A 1 2 sin2 A sin 2 A 2 sin A cos A 2 tan A 1 tan2 A

can be rewritten as sin2 A and


2

1 1 cos 2 A 2

1 cos A 1 cos 2 A. 2

EXAMPLE 1 Finding Function Values of 2 Given Information about


3 Given cos 5 and sin 0, nd sin 2, cos 2, and tan 2.

These identities are used to integrate the functions f A sin2 A and gA cos2 A.

Solution

To nd sin 2, we must rst nd the value of sin .

sin2

3 5

1 16 25 4 5

sin2 cos2 1; cos 3 5 Simplify. Choose the negative square root since sin 0.

sin2

sin

LIALMC07_0321227638.QXP

2/26/04

10:47 AM

Page 634

634 CHAPTER 7 Trigonometric Identities and Equations

Using the double-angle identity for sine, sin 2 2 sin cos 2 4 5


3 5

24 3 . sin 4 5 ; cos 5 25

Now we nd cos 2, using the rst of the double-angle identities for cosine. (Any of the three forms may be used.) cos 2 cos2 sin2 9 16 7 25 25 25

The value of tan 2 can be found in either of two ways. We can use the double4 sin 4 angle identity and the fact that tan cos 3 5 3. tan 2 2 tan 1 tan2
5 4 8 2 3 3 24 7 1 16 7 9 9 Simplify. (Section R.5)

Alternatively, we can nd tan 2 by nding the quotient of sin 2 and cos 2. tan 2 25 sin 2 24 7 7 cos 2 25
Now try Exercise 9.
24

EXAMPLE 2 Verifying a Double-Angle Identity

Verify that the following equation is an identity. cot x sin 2 x 1 cos 2 x


Solution

We start by working on the left side, using the hint from Section 7.1 about writing all functions in terms of sine and cosine. cot x sin 2 x cos x sin 2 x sin x
Quotient identity

cos x 2 sin x cos x Double-angle identity sin x


cos 2x 2 cos2 x 1, so 2 cos2 x 1 cos 2x

2 cos2 x 1 cos 2 x

The nal step illustrates the importance of being able to recognize alternative forms of identities.
Now try Exercise 27.

LIALMC07_0321227638.QXP

2/26/04

10:47 AM

Page 635

7.4 Double-Angle Identities and Half-Angle Identities

635

EXAMPLE 3 Simplifying Expressions Using Double-Angle Identities

Simplify each expression. (a) cos2 7x sin2 7x


Solution

(b) sin 15 cos 15

(a) This expression suggests one of the double-angle identities for cosine: cos 2 A cos2 A sin2 A. Substituting 7x for A gives cos2 7x sin2 7x cos 27x cos 14x. (b) If this expression were 2 sin 15 cos 15, we could apply the identity for sin 2 A directly since sin 2 A 2 sin A cos A. We can still apply the identity 1 with A 15 by writing the multiplicative identity element 1 as 2 2. sin 15 cos 15 1 2 sin 15 cos 15 2 1 2 sin 15 cos 15 2 1 sin2 15 2 1 sin 30 2 1 1 1 2 2 4
sin 30 1 2 (Section 5.3) Now try Exercises 13 and 15. Multiply by 1 in the form 1 2 2. Associative property (Section R.1) 2 sin A cos A sin 2 A, with A 15

Identities involving larger multiples of the variable can be derived by repeated use of the double-angle identities and other identities.

EXAMPLE 4 Deriving a Multiple-Angle Identity

Write sin 3x in terms of sin x.


Solution

sin 3x sin2x x sin 2x cos x cos 2x sin x 2 sin x cos x cos x cos2 x sin2 x sin x 2 sin x cos x cos x sin x sin x
2 2 3 2 2 3

Sine sum identity


(Section 7.3)

Double-angle identities Multiply.

2 sin x1 sin x 1 sin x sin x sin x cos2 x 1 sin2 x 2 sin x 2 sin3 x sin x sin3 x sin3 x 3 sin x 4 sin x
3

Distributive property
(Section R.1)

Combine terms. Now try Exercise 21.

LIALMC07_0321227638.QXP

2/26/04

10:47 AM

Page 636

636 CHAPTER 7 Trigonometric Identities and Equations

The next example applies a multiple-angle identity to answer a question about electric current.
EXAMPLE 5 Determining Wattage Consumption

If a toaster is plugged into a common household outlet, the wattage consumed is not constant. Instead, it varies at a high frequency according to the model W V2 , R

where V is the voltage and R is a constant that measures the resistance of the toaster in ohms. (Source: Bell, D., Fundamentals of Electric Circuits, Fourth Edition, Prentice-Hall, 1998.) Graph the wattage W consumed by a typical toaster with R 15 and V 163 sin 120 t in the window 0, .05 by 500, 2000. How many oscillations are there?
Solution For x = t, (163 sin 120t)2 W( t ) = 15 2000

Substituting the given values into the wattage equation gives W V 2 163 sin 120 t2 . R 15

To determine the range of W, we note that sin 120 t has maximum value 1, so 1632 the expression for W has maximum value 15 1771. The minimum value is 0. The graph in Figure 6 shows that there are six oscillations.
Now try Exercise 81.

0 500
Figure 6

.05

Product-to-Sum and Sum-to-Product Identities Because they make it possible to rewrite a product as a sum, the identities for cosA B and cosA B are used to derive a group of identities useful in calculus. Adding the identities for cosA B and cosA B gives
cosA B cos A cos B sin A sin B cosA B cos A cos B sin A sin B cosA B cosA B 2 cos A cos B or cos A cos B 1 cosA B cosA B. 2 1 cosA B cosA B. 2

Similarly, subtracting cosA B from cosA B gives


Looking Ahead to Calculus
The product-to-sum identities are used in calculus to nd integrals of functions that are products of trigonometric functions. One classic calculus text includes the following example: Evaluate

sin A sin B

Using the identities for sinA B and sinA B in the same way, we get two more identities. Those and the previous ones are now summarized.

Product-to-Sum Identities
cos A cos B sin A sin B 1 [cos(A B) cos(A B)] 2 1 [cos(A B) cos(A B)] 2
(continued)

cos 5x cos 3x dx.

The rst solution line reads: We may write cos 5x cos 3x 1 cos 8x cos 2x. 2

LIALMC07_0321227638.QXP

2/26/04

10:47 AM

Page 637

7.4 Double-Angle Identities and Half-Angle Identities

637

sin A cos B cos A sin B

1 [sin(A B) sin(A B)] 2 1 [sin(A B) sin(A B)] 2

EXAMPLE 6 Using a Product-to-Sum Identity

Write cos 2 sin as the sum or difference of two functions.


Solution

Use the identity for cos A sin B, with 2 A and B. cos 2 sin 1 sin2 sin2 2 1 1 sin 3 sin 2 2
Now try Exercise 41.

From these new identities we can derive another group of identities that are used to write sums of trigonometric functions as products.

Sum-to-Product Identities

sin A sin B 2 sin

sin A sin B 2 cos cos A cos B 2 cos

cos A cos B 2 sin


AB AB cos 2 2 AB AB sin 2 2 AB AB cos 2 2 AB AB sin 2 2

EXAMPLE 7 Using a Sum-to-Product Identity

Write sin 2 sin 4 as a product of two functions.


Solution

Use the identity for sin A sin B, with 2 A and 4 B.

sin 2 sin 4 2 cos

2 4 2 4 sin 2 2

2 cos

2 6 sin 2 2

2 cos 3 sin 2 cos 3 sin

sin sin (Section 7.1) Now try Exercise 45.

LIALMC07_0321227638.QXP

2/26/04

10:47 AM

Page 638

638 CHAPTER 7 Trigonometric Identities and Equations

Half-Angle Identities From the alternative forms of the identity for cos 2 A, A A A we derive three additional identities for sin 2 , cos 2 , and tan 2 . These are known as half-angle identities. A To derive the identity for sin 2 , start with the following double-angle identity for cosine and solve for sin x.
cos 2x 1 2 sin2 x 2 sin2 x 1 cos 2x sin x sin A 2
Add 2 sin2 x; subtract cos 2 x. Divide by 2; take square roots. (Section 1.4)

1 cos 2x 2 1 cos A 2

A Let 2x A, so x 2 ; substitute.

The sign in this identity indicates that the appropriate sign is chosen deA A pending on the quadrant of 2 . For example, if 2 is a quadrant III angle, we choose the negative sign since the sine function is negative in quadrant III. A We derive the identity for cos 2 using the double-angle identity 2 cos 2x 2 cos x 1. 1 cos 2x 2 cos2 x cos2 x 1 cos 2x 2
Add 1. Rewrite; divide by 2.

cos x cos An identity for tan A 2


A 2

1 cos 2x Take square roots. 2 1 cos A 2


Replace x with A 2.
A A

comes from the identities for sin 2 and cos 2 .

tan

A 2

sin A 2 cos A 2

1 cos A 2 1 cos A 2
A 2 A 2 A 2

1 cos A 1 cos A

We derive an alternative identity for tan tan A sin 2 cos tan


A 2 A 2

using double-angle identities.


Multiply by 2 cos 2 in numerator and denominator.
A

2 sin

cos
A 2

2 cos2

1 cos 2 A 2

sin 2 A 2

Double-angle identities

A sin A 2 1 cos A 1 cos A A . 2 sin A

A From this identity for tan 2 , we can also derive

tan

LIALMC07_0321227638.QXP

2/26/04

10:47 AM

Page 639

7.4 Double-Angle Identities and Half-Angle Identities

639

Half-Angle Identities
TEACHING TIP Point out that the
A follows 2 directly from the cosine and sine half-angle identities; however, the A other two identities for tan are 2 more useful. rst identity for tan

cos A 2

tan

A 2

1 cos A 2 tan

sin

A 2

1 cos A 2 tan A 1 cos A 2 sin A

1 cos A 1 cos A

A sin A 2 1 cos A

The last two identities for tan 2 do not require a sign choice. When using the other half-angle identities, select the plus or minus sign according to A the quadrant in which 2 terminates. For example, if an angle A 324, then A A A 2 162, which lies in quadrant II. In quadrant II, cos 2 and tan 2 are negative, A while sin 2 is positive.
NOTE

EXAMPLE 8 Using a Half-Angle Identity to Find an Exact Value

Find the exact value of cos 15 using the half-angle identity for cosine.
TEACHING TIP Have students compare the value of cos 15 in Example 8 to the value in Example 1(a) of Section 7.3, where we used the identity for the cosine of the difference of two angles. Although the expressions look completely different, they are equal, as suggested by a calculator approximation for both, .96592583.

Solution

cos 15 cos

1 30 2
3 1 2 2

1 cos 30 2
3 2 3 1 2 2 2 22

Choose the positive square root.

Simplify the radicals. (Section R.7) Now try Exercise 51.

EXAMPLE 9 Using a Half-Angle Identity to Find an Exact Value

Find the exact value of tan 22.5 using the identity tan
Solution

A 2

1
2

sin A cos A .

Since 22.5 2 45, replace A with 45. tan 22.5 tan

sin 45 45 2 2 1 cos 45 1 2 2 Now multiply numerator and denominator by 2. Then rationalize the denominator. tan 22.5 2 2 22 2 2 2 2 2 2 2 2 2 2 2 2 1 2 1 2
Now try Exercise 53.

LIALMC07_0321227638.QXP

2/26/04

10:47 AM

Page 640

640 CHAPTER 7 Trigonometric Identities and Equations


s 2

EXAMPLE 10

Finding Functions of
2 3,

Given Information about s


s s s

Given cos s
2

with

3 2

s 2, nd cos 2 , sin 2 , and tan 2 . 3 s 2 2

Solution

Since

s 2
0

and
s 2

3 s , 4 2

Divide by 2. (Section 1.7)


s

s
3 2 Figure 7

terminates in quadrant II. See Figure 7. In quadrant II, the values of cos 2 and s s tan 2 are negative and the value of sin 2 is positive. Now use the appropriate halfangle identities and simplify the radicals. s sin 2 cos

s 2


12 3 2 12 3 2
6

1 6 6 6 5 30 6 6

tan

s s sin 2 5 6 s 30 2 cos 2 5 6 s

Notice that it is not necessary to use a half-angle identity for tan 2 once we nd s s sin 2 and cos 2 . However, using this identity would provide an excellent check.
Now try Exercise 59.

EXAMPLE 11

Simplifying Expressions Using the Half-Angle Identities

Simplify each expression. (a)

1 cos 12x 2

(b)

1 cos 5 sin 5

Solution
A (a) This matches part of the identity for cos 2 .

cos Replace A with 12 x to get

A 2

1 cos A 2

1 cos 12x 12x cos cos 6x. 2 2


A 2

(b) Use the third identity for tan

given earlier with A 5 to get

1 cos 5 5 tan . sin 5 2


Now try Exercises 67 and 71.

LIALMC07_0321227638.QXP

2/26/04

10:47 AM

Page 641

7.4 Double-Angle Identities and Half-Angle Identities

641

7.4 Exercises
1. cos 25 5 ; sin 5 5

Use identities to nd values of the sine and cosine functions for each angle measure. See Example 1. 1. , given cos 2 2. , given cos 2 3 and terminates in quadrant I 5 3 and terminates in quadrant III 4 5 and x 12 2

14 2 ; sin 4 4 42 102 3. cos x ; sin x 12 12 30 6 4. cos x ; sin x 6 6 17 5. cos 2 ; 25 2. cos 421 25 119 120 6. cos 2 ; sin 2 169 169 4 3 7. cos 2x ; sin 2x 5 5 8 15 8. cos 2x ; sin 2x 17 17 39 9. cos 2 ; 49 455 sin 2 49 19 10. cos 2 ; 25 266 3 11. sin 2 25 2 2 3 3 12. 13. 14. 3 2 2 2 2 15. 16. 2 4 1 1 17. tan 102 18. tan 68 2 4 1 1 cos 94.2 20. sin 59 19. 4 16 3 21. cos 3x 4 cos x 3 cos x 22. sin 4x 4 sin x cos3 x 4 sin3 x cos x 3 tan x tan3 x 23. tan 3x 1 3 tan2 x 24. cos 4x 8 cos4 x 8 cos2 x 1 25. cos4 x sin4 x cos 2x 4 tan x cos2 x 2 tan x 26. sin 2x 1 tan2 x sin 2

3. x, given cos 2x 4. x, given cos 2x 5. 2, given sin

2 and x 3 2 2 and cos 0 5 12 and sin 0 13

6. 2, given cos

7. 2x, given tan x 2 and cos x 0 8. 2x, given tan x 5 and sin x 0 3 5 and cos 0 7

9. 2, given sin 10. 2, given cos

3 and sin 0 5

Use an identity to write each expression as a single trigonometric function value or as a single number. See Example 3. 11. cos2 15 sin2 15 14. 1 2 sin2 22 17. 20. tan 51 1 tan2 51 1 sin 29.5 cos 29.5 8 1 2 2 tan 15 1 tan2 15 1 15. 2 cos2 67 1 2 12. 18. tan 34 21 tan2 34 13. 1 2 sin2 15

1 8 2 1 1 19. sin2 47.1 4 2


16. cos2

Express each function as a trigonometric function of x. See Example 4. 21. cos 3x 22. sin 4x 23. tan 3x 24. cos 4x

Graph each expression and use the graph to conjecture an identity. Then verify your conjecture algebraically. 25. cos4 x sin4 x 26. 4 tan x cos2 x 2 tan x 1 tan2 x

Verify that each equation is an identity. See Example 2. 27. sin x cos x2 sin 2x 1 28. sec 2x sec2 x sec4 x 2 sec2 x sec4 x

LIALMC07_0321227638.QXP

2/26/04

10:47 AM

Page 642

642 CHAPTER 7 Trigonometric Identities and Equations 41. sin 160 sin 44 42. sin 225 sin 55 5 5 43. cos 5x cos x 2 2 1 1 44. cos x cos 9x 2 2 45. 2 sin 3x sin x 46. 2 cos 6.5x cos 1.5x 47. 2 sin 11.5 cos 36.5 48. 2 cos 98.5 sin 3.5 49. 2 cos 6x cos 2x 50. 2 cos 6x sin 3x 2 2 2 3 51. 52. 2 2 2 3 53. 54. 2 3 2 2 3 2 3 55. 56. 2 2 10 13 59. 60. 61. 3 4 4 50 206 62. 10 50 105 63. 10 50 1510 64. 10 5 65. 7 66. 5

29. sin 4x 4 sin x cos x cos 2x 31. 2 cos 2x cot x tan x sin 2x

30.

1 cos 2x cot x sin 2x

32. sin 4x 4 sin x cos x 8 sin3 x cos x 34. cos 2x 36. 1 tan2 x 1 tan2 x

33. sin 2x cos 2x sin 2x 4 sin3 x cos x 35. tan x cot x 2 csc 2x 37. sec2 39. sin2 x 2 2 1 cos x tan x sin x x 2 2 tan x

cot x tan x cos 2x cot x tan x x 1 cos x2 2 sin2 x

38. cot 2 40.

x x sin 2x cos2 sin2 2 sin x 2 2

Write each expression as a sum or difference of trigonometric functions. See Example 6. 41. 2 sin 58 cos 102 43. 5 cos 3x cos 2x 42. 2 cos 85 sin 140 44. sin 4x sin 5x

Write each expression as a product of trigonometric functions. See Example 7. 45. cos 4x cos 2x 48. sin 102 sin 95 46. cos 5x cos 8x 49. cos 4x cos 8x 47. sin 25 sin48 50. sin 9x sin 3x

Use a half-angle identity to nd each exact value. See Examples 8 and 9. 51. sin 67.5 54. tan 195 52. sin 195 55. cos 165 53. cos 195 56. sin 165

9 57.

Explain how you could use an identity of this section to nd the exact value of 1 sin 7.5. (Hint: 7.5 1 2 2 30.) the identity tan these answers are the same, without using a calculator. (Hint: If a 0 and b 0 and a2 b2, then a b.)
A 2 cos A cos A can be used to nd tan 22.5 3 22 , and sin A 1 cos A can be used to nd tan 22.5 2 1. Show that

1 58. The identity tan A 2 1

Find each of the following. See Example 10. 59. cos 60. sin 61. tan 62. 63. 64. 65. 66.

x 1 , given cos x , with 0 x 2 4 2


x 5 , given cos x , with x 2 8 2

3 , given sin , with 90 180 2 5 1 cos , given sin , with 180 270 2 5 x sin , given tan x 2, with 0 x 2 2 x cos , given cot x 3, with x 2 2 7 tan , given tan , with 180 270 2 3 5 cot , given tan , with 90 180 2 2

LIALMC07_0321227638.QXP

2/26/04

10:47 AM

Page 643

7.4 Double-Angle Identities and Half-Angle Identities

643

67. sin 20 68. cos 38 69. tan 73.5 70. cot 82.5 71. tan 29.87 72. tan 79.1 74. (a) tan is undened, so it 2 cannot be used. x sin 2 x (b) tan 2 cos x 2 75. 84 76. 106 77. 3.9 Rb 78. 2 79. (a) cos 2 R b (b) tan (c) 54 4 50 v 2 sin 2 80. (a) D 32 (b) approximately 35 ft


67. 70.

Use an identity to write each expression with a single trigonometric function. See Example 11. 1 cos 40 2 1 cos 165 1 cos 165 68. 71.

1 cos 76 2

69. 72.

1 cos 147 1 cos 147

1 cos 59.74 sin 59.74

sin 158.2 1 cos 158.2

sin A A 1 73. Use the identity tan A 2 1 cos A to derive the equivalent identity tan 2 by multiplying both the numerator and denominator by 1 cos A.

cos A sin A

74. Consider the expression tan 2 x .

(a) Why cant we use the identity for tanA B to express it as a function of x alone? sin (b) Use the identity tan cos to rewrite the expression in terms of sine and cosine. (c) Use the result of part (b) to show that tan 2 x cot x. (Modeling) Mach Number An airplane ying faster than sound sends out sound waves that form a cone, as shown in the gure. The cone intersects the ground to form a hyperbola. As this hyperbola passes over a particular point on the ground, a sonic boom is heard at that point. If is the angle at the vertex of the cone, then 1 , sin 2 m

where m is the Mach number for the speed of the plane. (We assume m 1.) The Mach number is the ratio of the speed of the plane and the speed of sound. Thus, a speed of Mach 1.4 means that the plane is ying at 1.4 times the speed of sound. In Exercises 75 78, one of the values or m is given. Find the other value. 3 5 75. m 76. m 77. 30 78. 60 2 4 (Modeling) Solve each problem. See Example 5. 79. Railroad Curves In the United States, circular railroad curves are designated by the degree of curvature, the central angle subtended by a chord of 100 ft. See the gure. (Source: Hay, W. W., Railroad Engineering, John Wiley & Sons, 1982.)
2.

b 50 50

(a) Use the gure to write an expression for cos (b) Use the result of part (a) and the third half-angle identity for tangent to write an expression for tan 4. (c) If b 12, what is the measure of angle to the nearest degree? 80. Distance Traveled by a Stone The distance D of an object thrown (or propelled) from height h (feet) at angle with initial velocity v is modeled by the formula

h D

v 2 sin cos v cos v sin 2 64h D . 32 See the gure. (Source: Kreighbaum, E. and K. Barthels, Biomechanics, Allyn & Bacon, 1996.) Also see the Chapter 5 Quantitative Reasoning. (a) Find D when h 0; that is, when the object is propelled from the ground. (b) Suppose a car driving over loose gravel kicks up a small stone at a velocity of 36 ft per sec (about 25 mph) and an angle 30. How far will the stone travel?

LIALMC07_0321227638.QXP

2/26/04

10:47 AM

Page 644

644 CHAPTER 7 Trigonometric Identities and Equations 81. a 885.6, c 885.6, 240 82. (a) For x = t, W = VI = (163 sin 120t)(1.23 sin 120t)
300

81. Wattage Consumption Refer to Example 5. Use an identity to determine values of a, c, and so that W a cost c. Check your answer by graphing both expressions for W on the same coordinate axes. 82. Amperage, Wattage, and Voltage Amperage is a measure of the amount of electricity that is moving through a circuit, whereas voltage is a measure of the force pushing the electricity. The wattage W consumed by an electrical device can be determined by calculating the product of the amperage I and voltage V. (Source: Wilcox, G. and C. Hesselberth, Electricity for Engineering Technology, Allyn & Bacon, 1970.) (a) A household circuit has voltage V 163 sin120 t when an incandescent lightbulb is turned on with amperage I 1.23 sin120 t. Graph the wattage W VI consumed by the lightbulb in the window 0, .05 by 50, 300. (b) Determine the maximum and minimum wattages used by the lightbulb. (c) Use identities to determine values for a, c, and so that W a cost c. (d) Check your answer by graphing both expressions for W on the same coordinate axes. (e) Use the graph to estimate the average wattage used by the light. For how many watts do you think this incandescent lightbulb is rated?

0 50

.05

(b) maximum: 200.49 watts; minimum: 0 watts (c) a 100.245, 240, c 100.245 (e) 100.245 watts

Summary Exercises on Verifying Trigonometric Identities


These summary exercises provide practice with the various types of trigonometric identities presented in this chapter. Verify that each equation is an identity. 1. tan cot sec csc 3. tan 5. x csc x cot x 2 2. csc cos2 sin csc 4. sec x sec x 6. 8. 10. 1 sin t 1 cos t sec t tan t 2 x tan2 1 1 cos x 2 1 1 2 cot t csc t sec t 1 sec t 1

sin t 1 cos t 1 cos t sin t 2 tan 1 tan2 2 cos2 1 sin cos

7. sin 2

9. cot tan 11. 13.

sinx y cot x cot y cosx y 1 cot x cot y sin tan tan 1 cos
x 1 tan2 2 2 x 1 tan 2

12. 1 tan2 14. 16. 18. 20. 22.

15. cos x 17.

tan2 t 1 tan t tan t csc2 t 2 tan 2 2 sec2 2

19. tan 4 21.

cot s tan s cos s sin s cos s sin s sin s cos s

2 cos 2 1 cos 1 cos2 x csc4 x cot 4 x 1 cos2 x 2 sec2 x cos 2x sec2 x 1 cos s sin s 2 csc s 1 cos s sin s x sec x tan x tan 2 4 tan cot 1 2 cos2 tan cot

LIALMC07_0321227638.QXP

2/26/04

10:47 AM

Page 645

7.5 Inverse Circular Functions 645

23. 25. 27.

tanx y tan y tan x 1 tanx y tan y cos4 x sin4 x 1 tan2 x cos2 x 2sin x sin3 x sin 2x cos x

24. 2 cos2 26. 28.

x tan x tan x sin x 2

csc t 1 sec t tan t2 csc t 1 1 x 1 x cot tan cot x 2 2 2 2

7.5 Inverse Circular Functions


Inverse Functions s Inverse Sine Function s Inverse Cosine Function Remaining Inverse Circular Functions s Inverse Function Values
s

Inverse Tangent Function

Inverse Functions We rst discussed inverse functions in Section 4.1. We give a quick review here for a pair of inverse functions f and f 1. 1. If a function f is one-to-one, then f has an inverse function f 1. 2. In a one-to-one function, each x-value corresponds to only one y-value and each y-value corresponds to only one x-value. 3. The domain of f is the range of f 1, and the range of f is the domain of f 1. 4. The graphs of f and f 1 are reections of each other about the line y x. 5. To nd f 1x from f x, follow these steps. Step 1 Replace f x with y and interchange x and y. Step 2 Solve for y. Step 3 Replace y with f 1x.
In the remainder of this section, we use these facts to develop the inverse circular (trigonometric) functions.
Looking Ahead to Calculus
The inverse circular functions are used in calculus to solve certain types of related-rates problems and to integrate certain rational functions.

Inverse Sine Function From Figure 8 and the horizontal line test, we see that y sin x does not dene a one-to-one function. If we restrict the domain to the interval 2 , 2 , which is the part of the graph in Figure 8 shown in color, this restricted function is one-to-one and has an inverse function. The range of y sin x is 1, 1, so the domain of the inverse function will be 1, 1, and its range will be 2 , 2 .
y

TEACHING TIP Mention that the inter-

val , contains enough of 2 2 the graph of the sine function to include all possible values of y. While other intervals could also be used, this interval is an accepted convention that is adopted by scientic calculators and graphing calculators.

2 2 3 2

,1 ( 2 ) (0, 0)
2

3 2 2 x

, 1) ( 2

1 2

y = sin x

Restricted domain , 2 2

Figure 8

LIALMC07_0321227638.QXP

2/26/04

10:47 AM

Page 646

646 CHAPTER 7 Trigonometric Identities and Equations


y 2

(1, 2) , ( 3 2 3) , ( 2 2 4) , (1 2 6)
(0, 0)
x 1

, 1 2 6

Reecting the graph of y sin x on the restricted domain across the line y x gives the graph of the inverse function, shown in Figure 9. Some key points are labeled on the graph. The equation of the inverse of y sin x is found by interchanging x and y to get x sin y. This equation is solved for y by writing y sin1 x (read inverse sine of x). As Figure 9 shows, the domain of y sin1 x is 1, 1, while the restricted domain of y sin x, 2 , 2 , is the 1 1 range of y sin x. An alternative notation for sin x is arcsin x.

, ) ( 2 2 4

(1, 2)
y=

, ) ( 3 2 3

Inverse Sine Function


y sin1 x or y arcsin x means that x sin y, for 2 y 2.

sin 1

x or y = arcsin x

Figure 9

We can think of y sin1 x or y arcsin x as y is the number in the inter val 2 , 2 whose sine is x. Just as we evaluated y log2 4 by writing it in exponential form as 2y 4 (Section 4.3), we can write y sin1 x as sin y x to evaluate it. We must pay close attention to the domain and range intervals.
EXAMPLE 1 Finding Inverse Sine Values

Find y in each equation. (a) y arcsin


Algebraic Solution

1 2

(b) y sin11
Graphing Calculator Solution

(c) y sin12

(a) The graph of the function dened by y arcsin x (Figure 9) includes the point 1 2 , 6 . Thus, arcsin 1 . 2 6

Alternatively, we can think of y arcsin 1 2 as y is 1 the number in whose sine is , 2 2 2 . Then 1 we can write the given equation as sin y 2 . 1 Since sin 6 2 and 6 is in the range of the arcsine function, y 6 . (b) Writing the equation y sin11 in the form sin y 1 shows that y 2 . This can be veri ed by noticing that the point 1, 2 is on the graph of y sin1 x. (c) Because 2 is not in the domain of the inverse sine function, sin12 does not exist.

To nd these values with a graphing calculator, we graph Y1 sin1 X and locate the points with X-values 1 1 2 and 1. Figure 10(a) shows that when X 2 , Y 6 .52359878. Similarly, Figure 10(b) shows that when X 1, Y 2 1.570796.
2 2

2
(a) Figure 10

2
(b)

Since sin12 does not exist, a calculator will give an error message for this input.
Now try Exercises 13 and 23.

CAUTION In Example 1(b), it is tempting to give the value of sin 1 as 3 3 3 2 , since sin 2 1. Notice, however, that 2 is not in the range of the inverse

sine function. Be certain that the number given for an inverse function value is in the range of the particular inverse function being considered.

LIALMC07_0321227638.QXP

2/26/04

10:47 AM

Page 647

7.5 Inverse Circular Functions 647

TEACHING TIP In problems like


Example 1(a), use phrases such as y is a value in radians between and whose sine is equal 2 2 1 to to help students understand 2 the meaning of inverse sine. Illustrate the answer graphically by 1 graphing y sin x and y in 2 the interval , , then 2 2 determine the point of intersection.

Our observations about the inverse sine function from Figure 9 lead to the following generalizations.

INVERSE SINE FUNCTION y sin1x or y arcsin x


Domain: 1, 1
y 2

Range: 2 , 2
2

y = sin 1 x

x 1
2 2

y 2 4 0
4 2

1
1 2 0 x 1

0
2 2

y = sin 1 x

2
Figure 11

The

inverse sine function is increasing and continuous on its domain 1, 1. Its x-intercept is 0, and its y-intercept is 0. Its graph is symmetric with respect to the origin; it is an odd function.

Inverse Cosine Function The function y cos1 x (or y arccos x) is dened by restricting the domain of the function y cos x to the interval 0, as in Figure 12, and then interchanging the roles of x and y. The graph of y cos1 x is shown in Figure 13. Again, some key points are shown on the graph.
y

( 1, )
y 1 (0, 1)

( )
2 0 1 2

,0 2

, 5 ) ( 3 2 6 , 3 ) ( 2 2 4 , 2 ( 1 2 3) 0,
x

(, 1) y = cos x
1

, (1 2 3) ( ) ( 2 , 2 4) , ( 3 2 6) 2
0 1 (1, 0)

Restricted domain [0, ]


Figure 12

y = cos 1 x

or y = arccos x

Figure 13

Inverse Cosine Function


y cos1 x or y arccos x means that x cos y, for 0 y .

LIALMC07_0321227638.QXP

2/26/04

10:47 AM

Page 648

648 CHAPTER 7 Trigonometric Identities and Equations


5 4

EXAMPLE 2 Finding Inverse Cosine Values

Find y in each equation. (a) y arccos 1


1 .5
5 4

(b) y cos1


2 2
2

Solution

(a) Since the point 1, 0 lies on the graph of y arccos x in Figure 13 on the previous page, the value of y is 0. Alternatively, we can think of y arccos 1 as y is the number in 0, whose cosine is 1, or cos y 1. Then y 0, since cos 0 1 and 0 is in the range of the arccosine function. (b) We must nd the value of y that satises cos y 2 , where y is in the interval 0, , the range of the function y cos1 x. The only value for y that 3 satises these conditions is 4 . Again, this can be veried from the graph in Figure 13.
Now try Exercises 15 and 21.

1 .5
These screens support the results of Example 2, since 3 2.3561945. 4

Our observations about the inverse cosine function lead to the following generalizations.

INVERSE COSINE FUNCTION y cos1 x or y arccos x


Domain: 1, 1
y

Range: 0,
y = cos 1 x

x 1
2 2

3 4 2 4

y = cos 1 x

0
2 2

x 1

1 0
Figure 14

The inverse cosine function is decreasing and continuous on its domain


1, 1. Its x-intercept is 1, and its y-intercept is 2. Its graph is not symmetric with respect to the y-axis or the origin.

Inverse Tangent Function Restricting the domain of the function y tan x to the open interval 2 , 2 yields a one-to-one function. By interchanging the roles of x and y, we obtain the inverse tangent function given by y tan1 x or y arctan x. Figure 15 shows the graph of the restricted tangent function. Figure 16 gives the graph of y tan1 x.

LIALMC07_0321227638.QXP

2/26/04

10:47 AM

Page 649

7.5 Inverse Circular Functions 649

y = tan x ,1 ( 4 )
y 2 1 2 x 2 1 (0, 0)

(0, 0)
2 0

(1, 4)
1 2

, 1) ( 4 Restricted domain , 2 2

, ) ( 3 3 6 1, ) ( 3 , 3) ( 4

, , (3 3 ) ( 3 3 6) x
2

y = tan 1 x or y = arctan x

Figure 15

Figure 16

Inverse Tangent Function


y tan1 x or y arctan x means that x tan y, for 2 y 2.

4 0 2

INVERSE TANGENT FUNCTION y tan1 x or y arctan x


Domain: ,
Range: 2 , 2

x
2

y 4 6 0
6 4

y 2 x 1 2

y = tan 1 x

1
3 3

y=
2

tan 1 x
1

0
3 3

0 2

2
Figure 17

4 0

The inverse tangent function is increasing and continuous on its domain


, . Its x-intercept is 0, and its y-intercept is 0. Its graph is symmetric with respect to the origin; it is an odd function. The lines y 2 and y 2 are horizontal asymptotes.

The first three screens show the graphs of the three remaining inverse circular functions. The last screen shows how they are defined.
Figure 18

Remaining Inverse Circular Functions The remaining three inverse trigonometric functions are dened similarly; their graphs are shown in Figure 18. All six inverse trigonometric functions with their domains and ranges are given in the table on the next page.

LIALMC07_0321227638.QXP

2/26/04

10:47 AM

Page 650

650 CHAPTER 7 Trigonometric Identities and Equations

Range Inverse Function y sin1 x y cos x y tan1 x y cot x y sec1 x y csc x


1 1 1

Domain 1, 1 1, 1 , , , 1 1, , 1 1,

Interval
2 , 2

Quadrants of the Unit Circle I and IV I and II I and IV I and II


2*

0,
2 , 2

0, 0, , y

I and II I and IV

2,2

, y

0*

Inverse Function Values The inverse circular functions are formally dened with real number ranges. However, there are times when it may be convenient to nd degree-measured angles equivalent to these real number values. It is also often convenient to think in terms of the unit circle and choose the inverse function values based on the quadrants given in the preceding table.
EXAMPLE 3 Finding Inverse Function Values (Degree-Measured Angles)

Find the degree measure of in the following. (a) arctan 1


Solution

(b) sec1 2

(a) Here must be in 90, 90, but since 1 0, must be in quadrant I. The alternative statement, tan 1, leads to 45. (b) Write the equation as sec 2. For sec1 x, is in quadrant I or II. Because 2 is positive, is in quadrant I and 60, since sec 60 2. Note that 60 the degree equivalent of 3 is in the range of the inverse secant function.
Now try Exercises 33 and 39.

The inverse trigonometric function keys on a calculator give results in the proper quadrant for the inverse sine, inverse cosine, and inverse tangent functions, according to the denitions of these functions. For example, on a calculator, in degrees, sin1 .5 30, sin1.5 30, tan11 45, and cos1.5 120. Finding cot 1 x, sec1 x, and csc1 x with a calculator is not as straightforward, because these functions must be expressed in terms of tan1 x, cos1 x, and sin1 x, respectively. If y sec1 x, for example, then sec y x, which must be written as a cosine function as follows: If sec y x, then 1 x cos y or cos y 1 , x and y cos1 1 . x

*The inverse secant and inverse cosecant functions are sometimes dened with different ranges. We use intervals that match their reciprocal functions (except for one missing point).

LIALMC07_0321227638.QXP

2/26/04

10:47 AM

Page 651

7.5 Inverse Circular Functions 651

In summary, to nd sec1 x, we nd cos1 x . Similar statements apply to csc1 x and cot 1 x. There is one additional consideration with cot 1 x. Since we take the inverse tangent of the reciprocal to nd inverse cotangent, the calculator gives values of inverse cotangent with the same range as inverse tangent, 2 , 2 , which is not the correct range for inverse cotangent. For inverse cotangent, the proper range must be considered and the results adjusted accordingly.
EXAMPLE 4 Finding Inverse Function Values with a Calculator

(a) Find y in radians if y csc13. (b) Find in degrees if arccot.3541.


Solution
1 (a) With the calculator in radian mode, enter csc13 as sin1 3 to get y .3398369095. See Figure 19.

(b) Set the calculator to degree mode. A calculator gives the inverse tangent value of a negative number as a quadrant IV angle. The restriction on the range of arccotangent implies that must be in quadrant II, so enter
Figure 19

arccot.3541 as tan1

1 180. .3541

As shown in Figure 19, 109.4990544.


Now try Exercises 43 and 49.

EXAMPLE 5 Finding Function Values Using Denitions of the Trigonometric Functions

Evaluate each expression without using a calculator.


13 3

(a) sin tan1


x

3 2

(b) tan cos1


5 13

Solution
3 (a) Let tan1 3 2 , so tan 2 . The inverse tangent function yields values only in quadrants I and IV, and since 3 2 is positive, is in quadrant I. Sketch in quadrant I, and label a triangle, as shown in Figure 20. By the Pythagorean theorem, the hypotenuse is 13. The value of sine is the quotient of the side opposite and the hypotenuse, so

2 = tan 1 3
2 Figure 20 y

sin tan1
12 13

A 5
0 x

5 5 (b) Let A cos1 13 . Since cos1 x for a negative . Then, cos A 13 value of x is in quadrant II, sketch A in quadrant II, as shown in Figure 21.


3 2

sin

3 313 . (Section 5.3) 13 13

tan cos1

A = cos 1 5

( 13 )


5 13

tan A

12 5

Figure 21

Now try Exercises 63 and 65.

LIALMC07_0321227638.QXP

2/26/04

10:47 AM

Page 652

652 CHAPTER 7 Trigonometric Identities and Equations

TEACHING TIP Point out that the


equations sinsin x x, coscos1x x, and tantan1x x are true wherever they are dened. However, sin1sin x x, cos1cos x x, and tan1tan x x are true only for values of x in the restricted domains of the sine, cosine, and tangent functions.
1

EXAMPLE 6 Finding Function Values Using Identities

Evaluate each expression without using a calculator. (a) cos arctan 3 arcsin
Solution

1 3

(b) tan 2 arcsin

2 5


1 3

1 (a) Let A arctan 3 and B arcsin 1 3 , so tan A 3 and sin B 3 . Sketch both A and B in quadrant I, as shown in Figure 22. Now, use the cosine sum identity.

cos arctan 3 arcsin


2 A

cosA B cos A cos B sin A sin B (Section 7.3) 1 3

cos arctan 3 cos arcsin

sin arctan 3 sin arcsin From Figure 22,


x

1 3

1 . 3

(1)

cos arctan 3 cos A sin arctan 3 sin A


3 B 1
x

1 , 2

cos arcsin sin arcsin

3 , 2

1 3 1 3

cos B sin B

22 , 3

Substitute these values into equation (1) to get cos arctan 3 arcsin

22
Figure 22

1 3

1 22 3 1 22 3 . 2 3 2 3 6

(b) Let arcsin 2 5 B. Then, from the double-angle tangent identity,


y

tan 2 arcsin

2 5

tan 2B

2 tan B . 1 tan2 B

(Section 7.4)

5 B
0

2
x

2 Since arcsin 2 5 B, sin B 5 . Sketch a triangle in quadrant I, nd the length of the third side, and then nd tan B. From the triangle in Figure 23, tan B 221 , and

21
Figure 23

2 tan 2 arcsin 5

2 21
2

2 21

4 21

4 21

421 . 17

Now try Exercises 69 and 75.

While the work shown in Examples 5 and 6 does not rely on a calculator, we can support our algebraic work with one. By entering cos arctan 3 arcsin 1 3 from Example 6(a) into a calculator, we get the approximation .1827293862, the same approximation as when we enter (the exact value we obtained 6 algebraically). Similarly, we obtain the same approximation when we evaluate tan 2 arcsin 2 5 and
421 17 , 22 3

supporting our answer in Example 6(b).

LIALMC07_0321227638.QXP

2/26/04

10:47 AM

Page 653

7.5 Inverse Circular Functions 653

EXAMPLE 7 Writing Function Values in Terms of u

Write each trigonometric expression as an algebraic expression in u. (a) sintan1 u


y

(b) cos2 sin1 u

Solution u2 + 1
1

u, u > 0
x

(a) Let tan1 u, so tan u. Here, u may be positive or negative. Since 2 tan1 u 2 , sketch in quadrants I and IV and label two triangles, as shown in Figure 24. Since sine is given by the quotient of the side opposite and the hypotenuse, sintan1 u sin u uu2 1 . u2 1 u2 1

u2 + 1

u, u < 0

The result is positive when u is positive and negative when u is negative.


Figure 24

(b) Let sin1 u, so sin u. To nd cos 2, use the identity cos 2 1 2 sin2 . cos2 sin1 u cos 2 1 2 sin2 1 2u2
Now try Exercises 83 and 85.

EXAMPLE 8 Finding the Optimal Angle of Elevation of a Shot Put

The optimal angle of elevation a shot-putter should aim for to throw the greatest distance depends on the velocity v of the throw and the initial height h of the shot. See Figure 25. One model for that achieves this greatest distance is

arcsin

v2 . 2v 2 64h

(Source: Townend, M. Stewart, Mathematics in Sport, Chichester, Ellis Horwood Limited, 1984.)

h D
Figure 25

Suppose a shot-putter can consistently throw the steel ball with h 6.6 ft and v 42 ft per sec. At what angle should he throw the ball to maximize distance?
Solution

To nd this angle, substitute and use a calculator in degree mode.

arcsin

422 2422 646.6

41.9 h 6.6, v 42
Now try Exercise 93.

LIALMC07_0321227638.QXP

2/26/04

10:47 AM

Page 654

654 CHAPTER 7 Trigonometric Identities and Equations

7.5 Exercises
1. one-to-one 2. range 3. domain 4. 1, ; y tan1 x or 4 y arctan x 5. 6. Sketch the reection of the graph of f across the line y x.

Concept Check

Complete each statement. . of y sin x. of y cos x.

1. For a function to have an inverse, it must be 2. The domain of y arcsin x equals the 3. The range of y cos1 x equals the


0,

, 2 2 (c) increasing (d) 2 is not in the domain. 8. (a) 1, 1 (b) 0, (c) decreasing 4 (d) is not in the range. 3 9. (a) , (b) , 2 2 (c) increasing (d) no 10. (a) , 1 1, ;
7. (a) 1, 1 (b)

4. The point 4 , 1 lies on the graph of y tan x. Therefore, the point lies on the graph of . 5. If a function f has an inverse and f 1, then f 11 6. How can the graph of f Concept Check
1

be sketched if the graph of f is known?

In Exercises 7 10, write short answers.

7. Consider the inverse sine function, dened by y sin1 x or y arcsin x. (a) What is its domain? (b) What is its range? (c) Is this function increasing or decreasing? (d) Why is arcsin2 not dened? 8. Consider the inverse cosine function, dened by y cos1 x or y arccos x. (a) What is its domain? (b) What is its range? (c) Is this function increasing or decreasing? 2 1 4 (d) Arccos 1 2 3 . Why is arccos 2 not equal to 3 ? 9. Consider the inverse tangent function, dened by y tan1 x or y arctan x. (a) What is its domain? (b) What is its range? (c) Is this function increasing or decreasing? (d) Is there any real number x for which arctan x is not dened? If so, what is it (or what are they)? 10. Give the domain and range of the three other inverse trigonometric functions, as dened in this section. (a) inverse cosecant function (c) inverse cotangent function 11. Concept Check (b) inverse secant function

, 0 0, 2 2 (b) , 1 1, ;

, 2 2 (c) , ; 0, 1 11. cos1 12. Find a 1 tan1 (or 180). a 13. 0 14. 15. 16. 4 4 17. 18. 19. 0 2 3 20. 21. 22. 3 2 4 2 5 23. 24. 25. 4 3 6 3 3 26. 27. 28. 4 4 4 5 29. 30. 31. 6 6 6 32. 33. 45 34. 120 4 35. 60 36. 45 37. 120

1 Is sec1 a calculated as cos1 1 a or as cos1 a ?

12. Concept Check For positive values of a, cot 1 a is calculated as tan1 1 a . How is cot 1 a calculated for negative values of a? Find the exact value of each real number y. Do not use a calculator. See Examples 1 and 2. 13. y sin1 0 16. y arctan1 19. y arctan 0 22. y tan11 14. y tan1 1 17. y sin11 20. y arcsin 23. y sin1 2 2 15. y cos11 18. y cos1 1 2


3 2

21. y arccos 0 24. y cos1

25. y arccos

28. y sec1 2 31. y arcsec 23 3


3 2

26. y arcsin


2 2


1 2

27. y cot 11 30. y arccot 3

29. y csc12 32. y csc1 2

LIALMC07_0321227638.QXP

2/26/04

10:47 AM

Page 655

7.5 Inverse Circular Functions 655

38. 41. 42. 43. 44. 45. 47. 49. 51. 53.

30 39. 120 40. 90 7.6713835 97.671207 113.500970 51.1691219 30.987961 46. 29.506181 .83798122 48. .96012698 2.3154725 50. 2.4605221 1.1900238 52. 1.1082303 y 54.
y 2 2 0 10 1 x 2 2 y = csc 1 x x
1 y = cot x

Give the degree measure of . Do not use a calculator. See Example 3. 33. arctan1 36. arcsin 34. arccos 37. cot 1

39. sec12


2 2

40. csc11


1 2 3 3

35. arcsin


3 2

38. csc12

Use a calculator to give each value in decimal degrees. See Example 4. 41. sin1.13349122 43. arccos.39876459 45. csc1 1.9422833 42. cos1.13348816 44. arcsin .77900016 46. cot 1 1.7670492

55.

y y = sec
1

56.
x 1

y y = arccsc 2 x 0 x 1 2

Use a calculator to give each real number value. (Be sure the calculator is in radian mode.) See Example 4. 47. y arctan 1.1111111 49. y cot .92170128 51. y arcsin .92837781
1

48. y arcsin .81926439 50. y sec11.2871684 52. y arccos .44624593

1 0 1

57.
2

y y = arcsec 1 x
2

Graph each inverse function as dened in the text. 53. y cot 1 x 56. y arccsc 2x 54. y csc1 x 57. y arcsec 1 x 2 55. y sec1 x

2 0

x 2

58. 1.003 is not in the domain of y sin1 x. 59. The domain of y tan1 x is , . 60. In both cases, the result is x. In each case, the graph is a straight line bisecting quadrants I and III (i.e., the line y x). 61. It is the graph of y x.
10

58. Explain why attempting to nd sin1 1.003 on your calculator will result in an error message. 59. Explain why you are able to nd tan1 1.003 on your calculator. Why is this situation different from the one described in Exercise 58?

Relating Concepts
For individual or collaborative investigation (Exercises 60 62)*
2 60. Consider the function dened by f x 3x 2 and its inverse f 1x x 3 . 1 1 Simplify f f x and f f x. What do you notice in each case? What would the graph look like in each case?

10

10

10

62. It does not agree because the range of the inverse tangent function is , , not 2 2 , , as was the case in Exercise 61.

61. Use a graphing calculator to graph y tantan1 x in the standard viewing window, using radian mode. How does this compare to the graph you described in Exercise 60? 62. Use a graphing calculator to graph y tan1tan x in the standard viewing window, using radian and dot modes. Why does this graph not agree with the graph you found in Exercise 61?

10

10

10 *The authors wish to thank Carol Walker of Hinds Community College for making a suggestion on which these exercises are based.

10

LIALMC07_0321227638.QXP

2/26/04

10:47 AM

Page 656

656 CHAPTER 7 Trigonometric Identities and Equations 7 15 5 64. 65. 3 4 5 56 120 7 66. 67. 68. 12 169 8 7 15 69. 70. 25 7 46 3 71. 72. 73. 2 25 5 63 16 74. 2 75. 76. 65 65 10 330 77. 20 48 253 78. 39 79. .894427191 80. .9682458366 81. .1234399811 82. .716386406 83. 1 u2 63. 1 u2 84. u 86. 88. 90. 1 u2 u uu2 9 u2 9 u2 5 u 85. 1 87. 89. 91. u2

Give the exact value of each expression without using a calculator. See Examples 5 and 6. 63. tan arccos

66. sec sin1 69. cos 2 arctan


3 4 1 5 4 3

64. sin arccos 67. sin 2 tan1 70. tan 2 cos1

1 4

12 5 1 4

65. costan12 68. cos 2 sin1 71. sin 2 cos1

72. cos2 tan12 75. cos tan1 77. sin sin1

5 3 tan1 12 4

1 tan13 2

73. secsec1 2

76. cos sin1 78. tan cos1

74. csc csc1 2 3 5 cos1 5 13

1 4 1 5

3 3 sin1 2 5

Use a calculator to nd each value. Give answers as real numbers. 79. costan1 .5 81. tanarcsin .12251014 80. sincos1 .25 82. cotarccos .58236841

u2 4 u u2 2

Write each expression as an algebraic (nontrigonometric) expression in u, u 0. See Example 7. 83. sinarccos u 86. cotarcsin u 89. tan sin1 84. tanarccos u 87. sin sec1 u u2 2

24 u2 4 u2

39 u2 92. 9 u2 93. (a) 45 (b) 45 94. (a) 113 (b) 84 (c) 60 (d) 47

91. sec arccot

4 u2 u

u 2

85. cosarcsin u 88. cos tan1

90. sec cos1

92. csc arctan

u2 5

9 u2 u


3 u

(Modeling) Solve each problem. 93. Angle of Elevation of a Shot Put Refer to Example 8. (a) What is the optimal angle when h 0? (b) Fix h at 6 ft and regard as a function of v. As v gets larger and larger, the graph approaches an asymptote. Find the equation of that asymptote. 94. Angle of Elevation of a Plane Suppose an airplane ying faster than sound goes directly over you. Assume that the plane is ying at a constant altitude. At the instant you feel the sonic boom from the plane, the angle of elevation to the plane is given by

2 arcsin

1 , m

where m is the Mach number of the planes speed. (The Mach number is the ratio of the speed of the plane and the speed of sound.) Find to the nearest degree for each value of m. (a) m 1.2 (b) m 1.5 (c) m 2 (d) m 2.5

LIALMC07_0321227638.QXP

2/26/04

10:47 AM

Page 657

7.5 Inverse Circular Functions 657

95. (a) 18 (b) 18 (c) 15 (e) 1.4142151 m (Note: Due to the computational routine, there may be a discrepancy in the last few decimal places.) y=
1

95. Observation of a Painting A painting 1 m high and 3 m from the oor will cut off an angle to an observer, where

tan1

tan 1

x x2 + 2

)
10

x . x2 2

Assume that the observer is x meters from the wall where the painting is displayed and that the eyes of x the observer are 2 m above the ground. (See the gure.) Find the value of for the following values of x. Round to the nearest degree. (a) 1 (b) 2 (c) 3 (d) Derive the formula given above. (Hint: Use the identity for tan . Use right triangles.) (e) Graph the function for with a graphing calculator, and determine the distance that maximizes the angle. (f) The idea in part (e) was first investigated in 1471 by the astronomer Regiomontanus. (Source: Maor, E., Trigonometric Delights, Princeton University Press, 1998.) If the bottom of the picture is a meters above eye level and the top of the picture is b meters above eye level, then the optimum value of x is ab meters. Use this result to find the exact answer to part (e). 96. Landscaping Formula A shrub is planted in a 100-ft-wide space between buildings measuring 75 ft and 150 ft tall. The location of the shrub determines how much sun it receives each day. Show that if is the angle in the gure and x is the distance of the shrub from the taller building, then the value of (in radians) is given by

0 .5 Radian mode

(f) 2 97. about 44.7%

arctan

75 150 . arctan 100 x x

150 ft 75 ft x 100 ft

97. Communications Satellite Coverage The gure shows a stationary communications satellite positioned 20,000 mi above the equator. What percent of the equator can be seen from the satellite? The diameter of Earth is 7927 mi at the equator.

LIALMC07_0321227638.QXP

2/26/04

10:47 AM

Page 658

658 CHAPTER 7 Trigonometric Identities and Equations

7.6 Trigonometric Equations


Solving by Linear Methods s Solving by Factoring s Solving by Quadratic Methods s Solving by Using Trigonometric Identities s Equations with Half-Angles s Equations with Multiple Angles s Applications Looking Ahead to Calculus
There are many instances in calculus where it is necessary to solve trigonometric equations. Examples include solving related-rates problems and optimization problems.

Earlier in this chapter, we studied trigonometric equations that were identities. We now consider trigonometric equations that are conditional; that is, equations that are satised by some values but not others.

Solving by Linear Methods Conditional equations with trigonometric (or circular) functions can usually be solved using algebraic methods and trigonometric identities.
EXAMPLE 1 Solving a Trigonometric Equation by Linear Methods

= 210 ' = 30
0 x

Solve 2 sin 1 0 over the interval 0, 360.


Solution

Because sin is the rst power of a trigonometric function, we use the same method as we would to solve the linear equation 2x 1 0. 2 sin 1 0 2 sin 1 sin 1 2
Subtract 1. (Section 1.1) Divide by 2.
1

(a)
y

= 330
0

' = 30

(b)
Figure 26

To nd values of that satisfy sin 2 , we observe that must be in either quadrant III or IV since the sine function is negative only in these two 1 quadrants. Furthermore, the reference angle must be 30 since sin 30 2 . The graphs in Figure 26 show the two possible values of , 210 and 330. The solution set is 210, 330. Alternatively, we could determine the solutions by referring to Figure 11 in Section 6.2 on page 546.
Now try Exercise 11.

Solving by Factoring
EXAMPLE 2 Solving a Trigonometric Equation by Factoring

Solve sin x tan x sin x over the interval 0, 360.


Solution

sin x tan x sin x sin x tan x sin x 0 sin x tan x 1 0 sin x 0 or x 180 tan x 1
Subtract sin x. Factor. (Section R.4)

tan x 1 0 Zero-factor property (Section 1.4) x 45 or x 225

x 0 or

The solution set is 0, 45, 180, 225.


Now try Exercise 31.

LIALMC07_0321227638.QXP

2/26/04

10:47 AM

Page 659

7.6 Trigonometric Equations 659

CAUTION There are four solutions in Example 2. Trying to solve the equation by dividing each side by sin x would lead to just tan x 1, which would give x 45 or x 225. The other two solutions would not appear. The missing solutions are the ones that make the divisor, sin x, equal 0. For this reason, we avoid dividing by a variable expression.

Solving by Quadratic Methods In Section 1.6, we saw that an equation in the form au2 bu c 0, where u is an algebraic expression, is solved by quadratic methods. The expression u may also be a trigonometric function, as in the equation tan2 x tan x 2 0.
EXAMPLE 3 Solving a Trigonometric Equation by Factoring

Solve tan2 x tan x 2 0 over the interval 0, 2.


Solution

This equation is quadratic in form and can be solved by factoring. tan2 x tan x 2 0 tan x 1 tan x 2 0 tan x 1 0 tan x 1 or or tan x 2 0 tan x 2
Factor. Zero-factor property

5 The solutions for tan x 1 over the interval 0, 2 are x 4 and x 4 . To solve tan x 2 over that interval, we use a scientic calculator set in radian mode. We nd that tan12 1.1071487. This is a quadrant IV number, based on the range of the inverse tangent function. (Refer to Figure 11 in Section 6.2 on page 546.) However, since we want solutions over the interval 0, 2, we must rst add to 1.1071487, and then add 2.

x 1.1071487 2.0344439 x 1.1071487 2 5.1760366 The solutions over the required interval form the solution set

, 4

5 , 4

2.0,

5.2

EXAMPLE 4 Solving a Trigonometric Equation Using the Quadratic Formula

Find all solutions of cot x cot x 3 1.


Solution

We multiply the factors on the left and subtract 1 to get the equation in standard quadratic form. cot 2 x 3 cot x 1 0 (Section 1.4)

Since this equation cannot be solved by factoring, we use the quadratic formula, with a 1, b 3, c 1, and cot x as the variable.

Exact values

Approximate values to the nearest tenth Now try Exercise 21.

LIALMC07_0321227638.QXP

2/26/04

10:47 AM

Page 660

660 CHAPTER 7 Trigonometric Identities and Equations

cot x

3 9 4 3 13 2 2

Quadratic formula with a 1, b 3, c 1


(Section 1.4)

cot x 3.302775638
TEACHING TIP Unlike the cosine
function, equations of the form y tan x will not contain a sec ond value for x between and 2 . Remind students that other 2 solutions to y tan x are found using period radians.

or cot x .3027756377 Use a calculator.

We cannot nd inverse cotangent values directly on a calculator, so we use the 1 fact that cot x tan x , and take reciprocals to get tan x .3027756377 x .2940013018 or tan x 3.302775638 or x 1.276795025.

To nd all solutions, we add integer multiples of the period of the tangent function, which is , to each solution found above. Thus, all solutions of the equation are written as .2940013018 n and 1.276795025 n, where n is any integer.*
Now try Exercise 43.

Solving by Using Trigonometric Identities Recall that squaring both sides of an equation, such as x 4 x 2, will yield all solutions but may also give extraneous values. (In this equation, 0 is a solution, while 3 is extraneous. Verify this.) The same situation may occur when trigonometric equations are solved in this manner.
EXAMPLE 5 Solving a Trigonometric Equation by Squaring

Solve tan x 3 sec x over the interval 0, 2.


TEACHING TIP Point out in
Example 5 that our rst goal is to rewrite the equation in terms of a single trigonometric function.

Solution Since the tangent and secant functions are related by the identity 1 tan2 x sec2 x, square both sides and express sec2 x in terms of tan2 x.

tan x 3 sec x tan x 23 tan x 3 sec2 x


2

x y2 x 2 2xy y 2
(Section R.3)
2

tan x 23 tan x 3 1 tan x


2

Pythagorean identity (Section 7.1) Subtract 3 tan2 x. Divide by 23; rationalize the denominator. (Section R.7)
5 6

23 tan x 2 tan x
TEACHING TIP Explain that whenever possible, answers should be given in exact form, such as 11 , rather than as decimal 6 approximations.

1 3 3 3
11 6 .

The possible solutions are Left side:

5 6

and

Now check them. Try

rst.

tan x 3 tan

5 23 3 3 3 6 3 3
(Section R.7)

Right side:

sec x sec

5 23 6 3

Not equal

*We usually give solutions of equations as solution sets, except when we ask for all solutions of a trigonometric equation.

LIALMC07_0321227638.QXP

2/26/04

10:47 AM

Page 661

7.6 Trigonometric Equations 661

y = tan x + 3 sec x
4

The check shows that Left side:

5 6

is not a solution. Now check tan sec

11 6 .

23 11 3 3 3 6 3 3 11 23 6 3
11 6

Right side:
4 Dot mode; radian mode The graph shows that on the interval [0, 2), the only x -intercept of the graph of y = tan x + 3 sec x is 5.7595865, which is an approximation for 11 , the sol6 ution found in Example 5.

Equal

This solution satises the equation, so

is the solution set.


Now try Exercise 41.

The methods for solving trigonometric equations illustrated in the examples can be summarized as follows.

Solving a Trigonometric Equation


1. Decide whether the equation is linear or quadratic in form, so you can determine the solution method. 2. If only one trigonometric function is present, rst solve the equation for that function. 3. If more than one trigonometric function is present, rearrange the equation so that one side equals 0. Then try to factor and set each factor equal to 0 to solve. 4. If the equation is quadratic in form, but not factorable, use the quadratic formula. Check that solutions are in the desired interval. 5. Try using identities to change the form of the equation. It may be helpful to square both sides of the equation rst. If this is done, check for extraneous solutions.

Some trigonometric equations involve functions of half-angles or multiple angles.

Equations with Half-Angles


EXAMPLE 6 Solving an Equation Using a Half-Angle Identity

Solve 2 sin

x 1 2 (b) give all solutions.

(a) over the interval 0, 2, and


Solution

(a) Write the interval 0, 2 as the inequality 0 x 2.


x The corresponding interval for 2 is

x . 2

Divide by 2. (Section 1.7)

LIALMC07_0321227638.QXP

2/26/04

10:47 AM

Page 662

662 CHAPTER 7 Trigonometric Identities and Equations y = 2 sin x 1 2

To nd all values of 2 over the interval 0, that satisfy the given equation, x rst solve for sin 2 . 2 sin x 1 2 1 x 2 2
Divide by 2.
1

sin
4 The x -intercepts are the solutions found in Example 6. Using Xscl = makes it 3 possible to support the exact solutions by counting the tick marks from 0 on the graph.

The two numbers over the interval 0, with sine value 2 are

and

5 6 ,

so

x 2 6
x

or or

x 5 2 6 x 5 . Multiply by 2. 3
5 3

The solution set over the given interval is 3 ,


TEACHING TIP As a slight variation
of the problem in Example 6, x replace with u, solve 2 2 sin u 1 for u, and then multiply the solutions by 2 to nd x.

(b) Since this is a sine function with period 4, all solutions are given by the expressions

5 4n and 4n, 3 3

where n is any integer.


Now try Exercise 63.

Equations with Multiple Angles


EXAMPLE 7 Solving an Equation with a Double Angle

Solve cos 2x cos x over the interval 0, 2.


Solution

First change cos 2x to a trigonometric function of x. Use the identity cos 2x 2 cos2 x 1 so the equation involves only cos x. Then factor. cos 2x cos x 2 cos x 1 cos x
2

Substitute; double-angle identity


(Section 7.4)

2 cos x cos x 1 0
2

Subtract cos x. Factor.

2 cos x 1 cos x 1 0 2 cos x 1 0 cos x 1 2 2 3 or or cos x 1

cos x 1 0 Zero-factor property

Over the required interval, x or x 4 3 or x 0.

4 The solution set is 0, 23 , 3 .

Now try Exercise 65.

LIALMC07_0321227638.QXP

2/26/04

10:47 AM

Page 663

7.6 Trigonometric Equations 663

CAUTION In the solution of Example 7, cos 2 x cannot be changed to cos x by dividing by 2 since 2 is not a factor of cos 2 x.

cos 2x 2

cos x

The only way to change cos 2 x to a trigonometric function of x is by using one of the identities for cos 2 x.

EXAMPLE 8 Solving an Equation Using a Multiple-Angle Identity

Solve 4 sin cos 3 over the interval 0, 360.


Solution

The identity 2 sin cos sin 2 is useful here. 4 sin cos 3 22 sin cos 3 4 2 2 2 sin 2 3 2 sin cos sin 2 (Section 7.4) sin 2 3 Divide by 2. 2

TEACHING TIP Students should be


aware that y sin ax may have as many as 2a solutions from 0 to 360.

From the given interval 0 360, the interval for 2 is 0 2 720. List all solutions over this interval. 2 60, 120, 420, 480 or

30, 60, 210, 240

Divide by 2.

The nal two solutions for 2 were found by adding 360 to 60 and 120, respectively, giving the solution set 30, 60, 210, 240.
Now try Exercise 83.

Applications Music is closely related to mathematics.


EXAMPLE 9 Describing a Musical Tone from a Graph
y = .004 sin(300 x)
.006

A basic component of music is a pure tone. The graph in Figure 27 models the sinusoidal pressure y P in pounds per square foot from a pure tone at time x t in seconds. (a) The frequency of a pure tone is often measured in hertz. One hertz is equal to one cycle per second and is abbreviated Hz. What is the frequency f in hertz of the pure tone shown in the graph? (b) The time for the tone to produce one complete cycle is called the period. Approximate the period T in seconds of the pure tone. (c) An equation for the graph is y .004 sin300 x. Use a calculator to estimate all solutions to the equation that make y .004 over the interval 0, .02.

.04

.006
Figure 27

LIALMC07_0321227638.QXP

2/26/04

10:47 AM

Page 664

664 CHAPTER 7 Trigonometric Identities and Equations Solution

Y2 = .004
.007

(a) From the graph in Figure 27 on the previous page, we see that there are 6 6 cycles in .04 sec. This is equivalent to .04 150 cycles per sec. The pure tone has a frequency of f 150 Hz. (b) Six periods cover a time of .04 sec. One period would be equal to 1 T .04 6 150 or .006 sec.

.02

(c) If we reproduce the graph in Figure 27 on a calculator as Y1 and also graph a second function as Y2 .004, we can determine that the approximate values of x at the points of intersection of the graphs over the interval 0, .02 are .0017, .0083, and .015. The rst value is shown in Figure 28.
Now try Exercise 87.

.007

Y1 = .004 sin(300 X)
Figure 28

A piano string can vibrate at more than one frequency when it is struck. It produces a complex wave that can mathematically be modeled by a sum of several pure tones. If a piano key with a frequency of f1 is played, then the corresponding string will not only vibrate at f1 but it will also vibrate at the higher frequencies of 2 f1, 3 f1, 4 f1, . . ., nf 1 , . . . . f1 is called the fundamental frequency of the string, and higher frequencies are called the upper harmonics. The human ear will hear the sum of these frequencies as one complex tone. (Source: Roederer, J., Introduction to the Physics and Psychophysics of Music, Second Edition, Springer-Verlag, 1975.)

EXAMPLE 10

Analyzing Pressures of Upper Harmonics

Suppose that the A key above middle C is played. Its fundamental frequency is f1 440 Hz, and its associated pressure is expressed as P1 .002 sin880 t. The string will also vibrate at f2 880, f3 1320, f4 1760, f5 2200, . . . Hz. .002 sin2640 t, 3 .002 sin4400 t. 5

The corresponding pressures of these upper harmonics are P2


P = P1 + P2 + P3 + P4 + P5
.005

.002 sin1760 t, 2 and

P3

P4 The graph of

.002 sin3520 t, 4

P5

.01

PP 1 P 2 P 3 P 4 P 5, shown in Figure 29, is saw-toothed. (a) What is the maximum value of P ?

.005
Figure 29

(b) At what values of x does this maximum occur over the interval 0, .01?

LIALMC07_0321227638.QXP

2/26/04

10:47 AM

Page 665

7.6 Trigonometric Equations 665

P = P1 + P2 + P3 + P4 + P5
.005

Solution

(a) A graphing calculator shows that the maximum value of P is approximately .00317. See Figure 30.
.01

(b) The maximum occurs at x .000188, .00246, .00474, .00701, and .00928. Figure 30 shows how the second value is found; the others are found similarly.
Now try Exercise 89. 1. Solve the linear equation for cot x. 2. Solve the linear equation for sin x. 3. Solve the quadratic equation for sec x by factoring. 4. Solve the quadratic equation for cos x by the zero-factor property. 5. Solve the quadratic equation for sin x using the quadratic formula. 6. Solve the quadratic equation for tan x using the quadratic formula. 7. Use an identity to rewrite as an equation with one trigonometric function.

.005
Figure 30

7.6 Exercises
8. Use an identity to rewrite as an equation with one trigonometric function. 4 , , 9. 10. 135, 180, 3 3 3 7 , 225, 270 11. 4 4 12. 14. 17. 18.

Concept Check Refer to the summary box on solving a trigonometric equation. Decide on the appropriate technique to begin the solution of each equation. Do not solve the equation. 1. 2 cot x 1 1 3. 5 sec2 x 6 sec x 5. 9 sin2 x 5 sin x 1 7. tan x cot x 0 Concept Check Answer each question. 2. sin x 2 3 4. 2 cos2 x cos x 1 6. tan2 x 4 tan x 2 0 8. cos2 x sin2 x 1

5 , 3 3

2 5 5 , , , 4 3 4 3

3 7 11 , , , 4 4 6 6

19. 20. 21. 22. 23. 24. 25. 26. 27. 28. 30. 31. 32. 33. 34. 35. 36.

7 3 11 , , 6 2 6 0, 2 4 , 3 3


13.

5 , 6 6

16. 0 15. 0

9. Suppose you are solving a trigonometric equation for solutions over the interval 0, 2, and your work leads to 2x 23 , 2, 83 . What are the corresponding values of x? 10. Suppose you are solving a trigonometric equation for solutions over the interval 0, 360, and your work leads to 1 3 45, 60, 75, 90. What are the corresponding values of ? Solve each equation for exact solutions over the interval 0, 2. See Examples 1 4. 11. 2 cot x 1 1 13. 2 sin x 3 4 15. tan2 x 3 0
2

3 11 , , , 6 2 2 6

12. sin x 2 3 14. 2 sec x 1 sec x 3 16. sec2 x 2 1 18. csc x 2 csc x 2 0 20. 2 cos2 x 3 cos x 0 22. 2 cos2 x cos x 1

17. cot x 1 3 cot x 1 0 19. cos x 2 cos x 1 0 21. 2 sin x 3 sin x 1


2

30, 210, 240, 300 0, 45, 225 90, 210, 330 60, 135, 240, 315 45, 135, 225, 315 0, 180 29. 45, 225 90, 270 0, 30, 150, 180 0, 90, 180, 270 0, 45, 135, 180, 225, 315 45, 135, 225, 315 53.6, 126.4, 187.9, 352.1 78.0, 282.0

Solve each equation for exact solutions over the interval 0, 360. See Examples 2 5. 23. cot 3 2 sin 3 0 25. 2 sin 1 csc 27. tan cot 0 29. csc2 2 cot 0 31. 2 tan sin tan 0
2 2

24. tan 1 cos 1 0 26. tan 1 3 3 cot 28. cos2 sin2 1 30. sin2 cos cos 32. sin2 cos2 0 34. cos2 sin2 0 36. 4 cos2 4 cos 1

33. sec2 tan 2 tan 35. 9 sin2 6 sin 1

LIALMC07_0321227638.QXP

2/26/04

10:47 AM

Page 666

666 CHAPTER 7 Trigonometric Identities and Equations 37. 149.6, 329.6, 106.3, 286.3 38. 38.4, 218.4, 104.8, 284.8 40. 68.5, 291.5 39. 0 41. 57.7, 159.2 42. 114.3, 335.7 43. .9 2n, 2.3 2n, 3.6 2n, 5.8 2n, where n is any integer 44. 2n, 2n, 3 5 2n, where n is any integer 3 2 45. 2n, 2n, 3 3 5 4 2n, 2n, where n 3 3 2 is any integer 46. 2n, 3 4 2n, where n is any integer 3 47. 33.6 360 n, 326.4 360 n, where n is any integer 48. 90 360 n, 221.8 360 n, 318.2 360 n, where n is any integer 49. 45 180 n, 108.4 180 n, where n is any integer 50. 135 360 n, 315 360 n, 71.6 360 n, 251.6 360 n, where n is any integer 51. .6806, 1.4159 52. 0, .3760 55. 56. 57. 58. 59. 60. 61. 62. 63.

37. tan2 4 tan 2 0 39. sin2 2 sin 3 0 41. cot 2 csc 3

38. 3 cot 2 3 cot 1 0 40. 2 cos2 2 cos 1 0 42. 2 sin 1 2 cos

Determine all solutions of each equation in radians (for x) or degrees (for ) to the nearest tenth as appropriate. See Example 4. 43. 3 sin2 x sin x 1 0 45. 4 cos2 x 1 0 47. 5 sec 6 sec
2

44. 2 cos2 x cos x 1 46. 2 cos2 x 5 cos x 2 0 48. 3 sin2 sin 2 50. sec2 2 tan 4

49.

2 tan 1 3 tan2

The following equations cannot be solved by algebraic methods. Use a graphing calculator to nd all solutions over the interval 0, 2. Express solutions to four decimal places. 51. x 2 sin x x 3 cos x 0 52. x 3 cos2 x 1 x1 2

9 53.

Explain what is wrong with the following solution. Solve tan 2 2 over the interval 0, 2. tan 2 2 2 tan 2 2 2 tan 1 5 or 4 4


11 13 23 , , , 12 12 12 12 2 4 5 , , , 3 3 3 3 7 11 , , 2 6 6
0,

9 54.

5 The solutions are 4 and 4 . x x The equation cot 2 csc 2 1 0 has no solution over the interval 0, 2. Using this information, what can be said about the graph of

y cot

x x csc 1 2 2

over this interval? Conrm your answer by actually graphing the function over the interval. Solve each equation for exact solutions over the interval 0, 2. See Examples 6 8. 55. cos 2x 3 2 56. cos 2x 1 2 57. sin 3x 1 60. cot 3x 3 63. sin x x 2 sin 2 2 x x cos 2 2

2 4 5 , , , , 3 3 3 3

7 13 19 25 31 , , , , , 18 18 18 18 18 18 7 13 19 25 31 , , , , , 18 18 18 18 18 18
3 5 11 13 , , , 8 8 8 8

58. sin 3x 0 61. 2 cos 2x 1 64. tan 4x 0 67. 8 sec2 70. sec x 4 2

59. 3 tan 3x 3 62. 23 sin 2x 3 65. sin x sin 2x 68. sin2 x 20 2

66. cos 2x cos x 0 69. sin

5 13 17 , , , 12 12 12 12 3 , 2 2

x x cos 2 2

LIALMC07_0321227638.QXP

2/26/04

10:47 AM

Page 667

7.6 Trigonometric Equations 667

64. 65. 66. 69.


0, 0, 0,

3 5 3 7 , , , , , , 4 2 4 4 2 4 5 , , 3 3

Solve each equation in Exercises 71 78 for exact solutions over the interval 0, 360. In Exercises 79 86, give all exact solutions. See Examples 6 8. 71. 2 sin 3 1 0 72. 2 cos 2 3 73. cos 76. 79. 82. 85.

2 4 , 3 3

68. 0 67. 0

70. 0

1 2 77. 2 sin 2 cos 2


74. sin 80. sin 2 2 cos2 83. 2 sin 2 4 sin 2 86. sin sin 2 0

71. 15, 45, 135, 165, 255, 285 72. 75, 105, 255, 285 73. 0 74. 180 75. 120, 240 76. 300 77. 30, 150, 270 78. 60, 90, 270, 300 79. 0 360 n, 30 360 n, 150 360 n, 180 360 n, where n is any integer 80. 45 360 n, 90 360 n, 225 360 n, 270 360 n, where n is any integer 81. 60 360 n, 300 360 n, where n is any integer 82. 70.5 360 n, 289.5 360 n, where n is any integer 83. 11.8 360 n, 78.2 360 n, 191.8 360 n, 258.2 360 n, where n is any integer 84. 22.5 360 n, 112.5 360 n, 202.5 360 n, 292.5 360 n, where n is any integer 85. 30 360 n, 90 360 n, 150 360 n, 210 360 n, 270 360 n, 330 360 n, where n is any integer 86. 0 360 n, 60 360 n, 180 360 n, 300 360 n, where n is any integer 87. (a) .00164 and .00355 (b) .00164, .00355 (c) outward 88. (a) 3 beats per sec For x = t, P(t) = .005 sin 440 t + .005 sin 446 t
.01

3 2 78. cos 1 cos 2 81. csc2 2 sec 2 84. 4 cos 2 8 sin cos
75. 23 sin

1 2 23 cos 3 2 1 sin cos 2 cos sin2 2 2 2 cos 2 1 cos 2

(Modeling) Solve each problem. See Examples 9 and 10. 87. Pressure on the Eardrum No musical instrument can generate a true pure tone. A pure tone has a unique, constant frequency and amplitude that sounds rather dull and uninteresting. The pressures caused by pure tones on the eardrum are sinusoidal. The change in pressure P in pounds per square foot on a persons eardrum from a pure tone at time t in seconds can be modeled using the equation P A sin2 f t , where f is the frequency in cycles per second, and is the phase angle. When P is positive, there is an increase in pressure and the eardrum is pushed inward; when P is negative, there is a decrease in pressure and the eardrum is pushed outward. (Source: Roederer, J., Introduction to the Physics and Psychophysics of Music, Second Edition, Springer-Verlag, 1975.) A graph of the tone middle C is shown in the gure. (a) Determine algebraically the values of t for which P 0 over 0, .005. (b) From the graph and your answer in part (a), determine the interval for which P 0 over 0, .005. (c) Would an eardrum hearing this tone be vibrating outward or inward when P 0?
For x = t, P( t ) = .004 sin 2 (261.63) t + 7
.005

.005

.005

88. Hearing Beats in Music Musicians sometimes tune instruments by playing the same tone on two different instruments and listening for a phenomenon known as beats. Beats occur when two tones vary in frequency by only a few hertz. When the two instruments are in tune, the beats disappear. The ear hears beats because the pressure slowly rises and falls as a result of this slight variation in the frequency. This phenomenon can be seen using a graphing calculator. (Source: Pierce, J., The Science of Musical Sound, Scientic American Books, 1992.) (a) Consider two tones with frequencies of 220 and 223 Hz and pressures P1 .005 sin 440 t and P2 .005 sin 446 t, respectively. Graph the pressure P P1 P2 felt by an eardrum over the 1-sec interval .15, 1.15. How many beats are there in 1 sec? (b) Repeat part (a) with frequencies of 220 and 216 Hz. (c) Determine a simple way to nd the number of beats per second if the frequency of each tone is given.

.15

1.15

.01

LIALMC07_0321227638.QXP

2/26/04

10:47 AM

Page 668

668 CHAPTER 7 Trigonometric Identities and Equations 88. (b) 4 beats per sec For x = t, P(t) = .005 sin 440 t + .005 sin 432 t
.01

89. Pressure of a Plucked String If a string with a fundamental frequency of 110 Hz is plucked in the middle, it will vibrate at the odd harmonics of 110, 330, 550, . . . Hz but not at the even harmonics of 220, 440, 660, . . . Hz. The resulting pressure P caused by the string can be modeled by the equation P .003 sin 220 t .003 .003 .003 sin 660 t sin 1100 t sin 1540t. 3 5 7

.15

1.15

.01

(Source: Benade, A., Fundamentals of Musical Acoustics, Dover Publications, 1990; Roederer, J., Introduction to the Physics and Psychophysics of Music, Second Edition, Springer-Verlag, 1975.) (a) Graph P in the window 0, .03 by .005, .005. (b) Use the graph to describe the shape of the sound wave that is produced. (c) See Exercise 87. At lower frequencies, the inner ear will hear a tone only when the eardrum is moving outward. Determine the times over the interval 0, .03 when this will occur. 90. Hearing Difference Tones Small speakers like those found in older radios and telephones often cannot vibrate slower than 200 Hzyet 35 keys on a piano have frequencies below 200 Hz. When a musical instrument creates a tone of 110 Hz, it also creates tones at 220, 330, 440, 550, 660, . . . Hz. A small speaker cannot reproduce the 110-Hz vibration but it can reproduce the higher frequencies, which are called the upper harmonics. The low tones can still be heard because the speaker produces difference tones of the upper harmonics. The difference between consecutive frequencies is 110 Hz, and this difference tone will be heard by a listener. We can model this phenomenon using a graphing calculator. (Source: Benade, A., Fundamentals of Musical Acoustics, Dover Publications, 1990.) (a) In the window 0, .03 by 1, 1, graph the upper harmonics represented by the pressure

(c) The number of beats is equal to the absolute value of the difference in the frequencies of the two tones. 89. (a) For x = t, P(t) = .003 sin 220 t +
.003 3 .003 5 .003 7

sin 660 t + sin 1100 t + sin 1540 t

.005

.03

.005

(b) The graph is periodic, and the wave has jagged square tops and bottoms. (c) This will occur when t is in one of these intervals: .0045, .0091, .0136, .0182, .0227, .0273. 90. (a) For x = t, P(t) =
1 sin[2 (220)t] 2 1 sin[2 (330)t] 3 1 sin[2 (440)t] 4

1 1 1 sin2 220t sin2 330t sin2 440t. 2 3 4 (b) Estimate all t-coordinates where P is maximum. (c) What does a person hear in addition to the frequencies of 220, 330, and 440 Hz? (d) Graph the pressure produced by a speaker that can vibrate at 110 Hz and above. P 91. Daylight Hours in New Orleans The seasonal variation in length of daylight can be modeled by a sine function. For example, the daily number of hours of daylight in New Orleans is given by h 7 2 x 35 sin , 3 3 365

+ +

where x is the number of days after March 21 (disregarding leap year). (Source: Bushaw, Donald et al., A Sourcebook of Applications of School Mathematics. Copyright 1980 by The Mathematical Association of America.)
.03

(a) On what date will there be about 14 hr of daylight? (b) What date has the least number of hours of daylight? (c) When will there be about 10 hr of daylight? (Modeling) Alternating Electric Current The study of alternating electric current requires the solutions of equations of the form i I max sin 2 ft, for time t in seconds, where i is instantaneous current in amperes, I max is maximum current in amperes, and f is the number of cycles per second. (Source: Hannon, R. H., Basic

(b) .0007576, .009847, .01894, .02803 (c) 110 Hz

LIALMC07_0321227638.QXP

2/26/04

10:48 AM

Page 669

7.6 Trigonometric Equations 669

90. (d) For x = t, P(t) = sin[2 (110)t] +


1 2 1 3 1 4

Technical Mathematics with Calculus, W. B. Saunders Company, 1978.) Find the smallest positive value of t, given the following data. 92. i 40, I max 100, f 60 94. i I max, f 60 (Modeling) Solve each problem. 96. Accident Reconstruction The model 93. i 50, I max 100, f 120 95. i 1 I max, f 60 2

sin[2 (220)t] + sin[2 (330)t] + sin[2 (440)t]

.03

.342D cos h cos2

16D 2 V 02

91. (a) 91.3 days after March 21, on June 20 (b) 273.8 days after March 21, on December 19 (c) 228.7 days after March 21, on November 4, and again after 318.8 days, on February 2 92. .001 sec 93. .0007 sec 94. .004 sec 95. .0014 sec 1 96. 14 97. (a) sec 4 1 (b) sec (c) .21 sec 6 1 98. (a) 2 sec (b) 3 sec 3 99. (a) One such value is . 3 (b) One such value is . 4

is used to reconstruct accidents in which a vehicle vaults into the air after hitting an obstruction. V0 is velocity in feet per second of the vehicle when it hits, D is distance (in feet) from the obstruction to the landing point, and h is the difference in height (in feet) between landing point and takeoff point. Angle is the takeoff angle, the angle between the horizontal and the path of the vehicle. Find to the nearest degree if V0 60, D 80, and h 2. 97. Electromotive Force In an electric circuit, let V cos 2 t model the electromotive force in volts at t seconds. Find the smallest positive value of t where 0 t 1 2 for each value of V. (a) V 0 (b) V .5 (c) V .25 A coil of wire rotating in a magnetic eld 98. Voltage Induced by a Coil of Wire induces a voltage modeled by e 20 sin

t , 4 2

where t is time in seconds. Find the smallest positive time to produce each voltage. (a) 0 (b) 103 99. Movement of a Particle A particle moves along a straight line. The distance of the particle from the origin at time t is modeled by st sin t 2 cos t. Find a value of t that satises each equation. (a) st 2 3 2 (b) st 32 2

9 100.

Explain what is wrong with the following solution for all x over the interval 0, 2 of the equation sin2 x sin x 0. sin2 x sin x 0 sin x 1 0 sin x 1
Divide by sin x. Add 1.

x 2
The solution set is 2 .

LIALMC07_0321227638.QXP

2/26/04

10:48 AM

Page 670

670 CHAPTER 7 Trigonometric Identities and Equations

7.7 Equations Involving Inverse Trigonometric Functions


Solving for x in Terms of y Using Inverse Functions
s

Solving Inverse Trigonometric Equations

Until now, the equations in this chapter have involved trigonometric functions of angles or real numbers. Now we examine equations involving inverse trigonometric functions.

Solving for x in Terms of y Using Inverse Functions


EXAMPLE 1 Solving an Equation for a Variable Using Inverse Notation

Solve y 3 cos 2x for x.


Solution We want cos 2 x alone on one side of the equation so we can solve for 2 x, and then for x.

y 3 cos 2x y cos 2x 3 2x arccos x y 3


Divide by 3. Denition of arccosine (Section 7.5) Multiply by 2 . Now try Exercise 7.
1

1 y arccos 2 3

Solving Inverse Trigonometric Equations


EXAMPLE 2 Solving an Equation Involving an Inverse Trigonometric Function

Solve 2 arcsin x .
Solution

First solve for arcsin x, and then for x. 2 arcsin x arcsin x

2 2

Divide by 2. Denition of arcsine (Section 7.5)


(Section 6.2)

x sin x1

Verify that the solution satises the given equation. The solution set is 1.
Now try Exercise 23.

LIALMC07_0321227638.QXP

2/26/04

10:48 AM

Page 671

7.7 Equations Involving Inverse Trigonometric Functions 671

EXAMPLE 3 Solving an Equation Involving Inverse Trigonometric Functions

Solve cos1 x sin1


Solution

1 . 2

1 Let sin1 1 2 u. Then sin u 2 and for u in quadrant I, the equation

becomes
2 u
0

1
x

cos1 x u cos u x. Alternative form Sketch a triangle and label it using the facts that u is in quadrant I and sin u 1 2. See Figure 31. Since x cos u, x
3 2 , 3 and the solution set is 2 . Check.

Figure 31

Now try Exercise 29.

EXAMPLE 4 Solving an Inverse Trigonometric Equation Using an Identity

Solve arcsin x arccos x


Solution

. 6

Isolate one inverse function on one side of the equation.

arcsin x arccos x

6 6
Add arccos x. (1)

sin arccos x

arcsin x arccos x

6 6 6

Denition of arcsine

Let u arccos x, so 0 u by denition. sin u sin u

x sin u cos

Substitute.

(2)

cos u sin 6 6

Sine sum identity (Section 7.3)

Substitute this result into equation (2) to get sin u cos


y

cos u sin x. (3) 6 6 6 2 . 3

From equation (1) and by the denition of the arcsine function,

1 u
0

1 x2
x
x

arccos x 2 2 arccos x 3

Subtract 6 . (Section 1.7)

Figure 32

Since 0 arccos x , we must have 0 arccos x 3 . Thus, x 0, and we can sketch the triangle in Figure 32. From this triangle we nd that sin u 1 x 2.

LIALMC07_0321227638.QXP

2/26/04

10:48 AM

Page 672

672 CHAPTER 7 Trigonometric Identities and Equations


1 Now substitute into equation (3) using sin u 1 x 2, sin 6 2 , 3 cos 6 2 , and cos u x.

sin u cos

cos u sin 6 3 1 x2 2 x

x 6 1 x 2

(3)

1 x2 3 x 2x 3 1 x2 x
31 x x
2 2 2 2

Multiply by 2. Subtract x. Square both sides. (Section 1.6) Distributive property (Section R.1) Add 3x 2.

3 3x x
2

3 4x x x

3 2

3 4

Solve for x; Choose the positive square root because x 0. Quotient rule (Section R.7)

3 To check, replace x with 2 in the original equation:

arcsin

3 3 arccos , 2 2 3 6 6
3

as required. The solution set is 2 .


Now try Exercise 31.

7.7 Exercises
1. C 5. 6. 7. 8. 9. 2. A 3. C 4. C y x arccos 5 x arcsin 4y 1 arccot 2y x 3 x arcsec 12y 1 y x arctan 2 3

Concept Check

Answer each question. B. arccos 0 x B. arccos C. arcsin 0 x x 3 x 3

1. Which one of the following equations has solution 0? 2. Which one of the following equations has solution 4? A. arcsin 2 x 2 A. arctan 1 x

3. Which one of the following equations has solution 34 ?


2 2

C. arctan

A. arctan 1 x 3 x 3

B. arcsin

2 x 2

C. arccos

4. Which one of the following equations has solution 6? A. arctan B. arccos


1 2

C. arcsin


2 2 1 2

LIALMC07_0321227638.QXP

2/26/04

10:48 AM

Page 673

7.7 Equations Involving Inverse Trigonometric Functions 673

10. x 2 arcsin 11. 12. 13. 14. 15. 16. 17. 18. 19.

20. x arccos 23. 22

25. 3 26. 27. 30. 33. 35. 3 5 3 4 1 2 28.


y4 3 24. 0 33 2 6 29. 12 5 4 5 31. 0 32. 34. 0 36. 3 2 1 2 5 5
2 2

y 3 y x 4 arccos 6 x 3 arcsiny y 1 arccos x 5 2 y 1 arccot x 5 3 x 3 arccos y 1 x 1 arctan y 2 x arcsin y 2 x arccot y 1 y4 x arcsin 2

Solve each equation for x. See Example 1. 5. y 5 cos x 1 sec x 8. 6y 2 11. y 6 cos x 4 6. 4y sin x 9. y 3 tan 2x 12. y sin x 3 7. 2y cot 3x 10. y 3 sin x 2

13. y 2 cos 5x 16. y tan2x 1 19. y 2 sin x 4

14. y 3 cot 5x 17. y sin x 2 20. y 4 3 cos x

15. y cosx 3 18. y cot x 1

9 21.

9 22.

Refer to Exercise 17. A student attempting to solve this equation wrote as the rst step y sinx 2, inserting parentheses as shown. Explain why this is incorrect. Explain why the equation sin1 x cos1 2 cannot have a solution. (No work is required.)

Solve each equation for exact solutions. See Examples 2 and 3. 23. 4 y cos1 3 4 24. 4 4 tan1 y 26. arccos y

25. 2 arccos


3 5

y 2 3 3 4

5 13

27. arcsin x arctan 29. cos1 x sin1

28. arctan x arccos 30. cot 1 x tan1 4 3

Solve each equation for exact solutions. See Example 4. 31. sin1 x tan1 1 33. 35.

37. 0 38. 0 39. Y = arcsin X arccos X 6 2

37. 39. 40.

40. Y1 = arcsin X arccos X Y2 = 6 2


2 2

2 32. sin1 x tan1 3 4 3 3 3 34. arccos x 2 arcsin arccos x 2 arcsin 2 2 3 36. arcsin 2x arcsin x arcsin 2x arccos x 6 2 38. sin1 x tan1 x 0 cos1 x tan1 x 2 Provide graphical support for the solution in Example 4 by showing that the graph 3 of y arcsin x arccos x 6 has x-intercept 2 .8660254. Provide graphical support for the solution in Example 4 by showing that the x-coordinate of the point of intersection of the graphs of Y1 arcsin X arccos X 3 and Y2 6 is 2 .8660254.

41. 4.4622037 42. 2.2824135

The following equations cannot be solved by algebraic methods. Use a graphing calculator to nd all solutions over the interval 0, 6. Express solutions to as many decimal places as your calculator displays. 41. arctan x3 x 2 0 42. sin1.2x 3 x

LIALMC07_0321227638.QXP

2/26/04

10:48 AM

Page 674

674 CHAPTER 7 Trigonometric Identities and Equations

(Modeling) Solve each problem. 43. (a) A .00506, .484; P .00506 sin440 t .484 43. Tone Heard by a Listener When two sources located at different positions pro(b) The two graphs are the same. duce the same pure tone, the human ear will often hear one sound that is equal to the For x = t, sum of the individual tones. Since the sources are at different locations, they will P ( t ) = .00506 sin(440t + .484) have different phase angles . If two speakers located at different positions produce P1( t ) + P2( t ) = .0012 sin(440t + .052) + pure tones P1 A 1 sin2 ft 1 and P2 A 2 sin2 ft 2, where 1, 4 .004 sin(440t + .61) 2 , then the resulting tone heard by a listener can be written as
.006

P A sin2 ft , where

A A 1 cos 1 A 2 cos 22 A 1 sin 1 A 2 sin 22


0 .01

and
.006

arctan

A 1 sin 1 A 2 sin 2 . A 1 cos 1 A 2 cos 2

(Source: Fletcher, N. and T. Rossing, The Physics of Musical Instruments, Second Edition, Springer-Verlag, 1998.) (a) Calculate A and if A 1 .0012, 1 .052, A 2 .004, and 2 .61. Also nd an expression for P A sin2 ft if f 220. (b) Graph Y1 P and Y2 P1 P2 on the same coordinate axes over the interval 0, .01. Are the two graphs the same?

44. (a) A .0035, .470; P .0035 sin600 t .47 (b) The two graphs are the same. For x = t, P ( t ) = .0035 sin(600t + .47)

P1( t ) + P2( t ) = .0025 sin 600t + + 7 45. Depth of Field When a large-view camera is used to take a picture of an object that is not parallel to the lm, the lens board should be tilted so that the planes contain.001 sin 600t + 6

44. Tone Heard by a Listener Repeat Exercise 43, with A 1 .0025, 1 7, A 2 .001, 2 6 , and f 300.

.006

.01

ing the subject, the lens board, and the lm intersect in a line. This gives the best depth of eld. See the gure. (Source: Bushaw, Donald et al., A Sourcebook of Applications of School Mathematics. Copyright 1980 by The Mathematical Association of America.)
Subject Lens y x Film z

.006

x xy 45. (a) tan ; tan z z xy x (b) tan tan (c) arctan

(d) arctan


x tan xy x y tan x
x x

46. (a) x sin u, (b)


y

u 2 2

(a) Write two equations, one relating , x, and z, and the other relating , x, y, and z. (b) Eliminate z from the equations in part (a) to get one equation relating , , x, and y. (c) Solve the equation from part (b) for . (d) Solve the equation from part (b) for . 46. Programming Language for Inverse Functions In Visual Basic, a widely used programming language for PCs, the only inverse trigonometric function available is arctangent. The other inverse trigonometric functions can be expressed in terms of arctangent as follows. y (a) Let u arcsin x. Solve the equation for x in terms of u. (b) Use the result of part (a) to label the three sides of the triangle in the gure in terms of x. (c) Use the triangle from part (b) to write an equation for tan u in terms of x. (d) Solve the equation from part (c) for u.

1 u 0

1 x2

x1 x 2 (c) tan u 1 x2

u
0

LIALMC07_0321227638.QXP

2/26/04

10:48 AM

Page 675

7.7 Equations Involving Inverse Trigonometric Functions 675

x1 x 2 1 x2 e 1 47. (a) t arcsin 2 f E max (b) .00068 sec 48. (b) (i) approximately .94 or 4.26 (ii) approximately .60 or 6.64 (c) (i) approximately .54 (ii) approximately .61 3 49. (a) t arcsin 3y 4 (b) .27 sec 50. y = sec 1 x (d) u arctan

47. Alternating Electric Current In the study of alternating electric current, instantaneous voltage is modeled by e E max sin 2 ft, where f is the number of cycles per second, E max is the maximum voltage, and t is time in seconds. (a) Solve the equation for t. (b) Find the smallest positive value of t if E max 12, e 5, and f 100. Use a calculator. 48. Viewing Angle of an Observer While visiting a museum, Marsha Langlois views a painting that is 3 ft high and hangs 6 ft above the ground. See the gure. Assume her eyes are 5 ft above the ground, and let x be the distance from the spot where she is standing to the wall displaying the painting. (a) Show that , the viewing angle subtended by the painting, is given by

3 ft

6 ft

5 ft

4 0 Radian mode

x ft

tan1

4 x

tan1

1 . x

(b) Find the value of x for each value of .

(ii) 6 8 (c) Find the value of for each value of x. (i) x 4 (ii) x 3 49. Movement of an Arm In the exercises for Section 6.3 we found the equation
(i) y 4 t 1 , sin 3 3

where t is time (in seconds) and y is the angle formed by a rhythmically moving arm. (a) Solve the equation for t. (b) At what time(s) does the arm form an angle of .3 radian? 50. The function y sec1 x is not found on graphing calculators. However, with some models it can be graphed as y

x 0 x 0 tan1 x 2 1 . 2 2

(This formula appears as Y1 in the screen here.) Use the formula to obtain the graph of y sec1 x in the window 4, 4 by 0, .

LIALMC07_0321227638.QXP

2/26/04

10:48 AM

Page 676

676 CHAPTER 7 Trigonometric Identities and Equations

Chapter 7 Summary
NEW SYMBOLS
sin1 x (arcsin x) inverse sine of x cos1 x (arccos x) inverse cosine of x tan1 x (arctan x) inverse tangent of x cot 1 x (arccot x) inverse cotangent of x sec1 x (arcsec x) inverse secant of x csc1 x (arccsc x) inverse cosecant of x

QUICK REVIEW
CONCEPTS EXAMPLES

7.1 Fundamental Identities

Reciprocal Identities cot 1 tan sec 1 cos csc 1 sin

3 If is in quadrant IV and sin 5 , nd csc , cos , and sin.

Quotient Identities tan sin cos cot cos sin

csc

1 1 5 sin 3 3 5

Pythagorean Identities sin2 cos2 1 tan2 1 sec2 1 cot 2 csc2 Negative-Angle Identities sin sin csc csc cos cos sec sec tan tan cot cot


3 5

sin2 cos2 1
2

cos2 1 cos2 1 cos 16 9 25 25 4 16 25 5 3 5


cos is positive in quadrant IV.

sin sin

7.2 Verifying Trigonometric Identities

See the box titled Hints for Verifying Identities on page 613.
7.3 Sum and Difference Identities

Cofunction Identities cos90 sin sin90 cos tan90 cot Sum and Difference Identities cosA B cos A cos B sin A sin B cosA B cos A cos B sin A sin B sinA B sin A cos B cos A sin B sinA B sin A cos B cos A sin B cot90 tan sec90 csc csc90 sec

Find a value of such that tan cot 78. tan cot 78 cot90 cot 78 90 78

12
Find the exact value of cos15. cos15 cos30 45 cos 30 cos 45 sin 30 sin 45 1 2 3 2 2 2 2 2 6 2 6 2 4 4 4

LIALMC07_0321227638.QXP

2/26/04

10:48 AM

Page 677

CHAPTER 7

Summary 677

CONCEPTS

EXAMPLES

Sum and Difference Identities tanA B tanA B tan A tan B 1 tan A tan B tan A tan B 1 tan A tan B

Write tan 4 in terms of tan .

tan

tan 4 tan 4 1 tan 4 tan 1 tan tan 4 1 1 tan

7.4 Double-Angle Identities and Half-Angle Identities

Double-Angle Identities cos 2A cos2 A sin2 A cos 2A 2 cos A 1


2

5 Given cos 13 and sin 0, nd sin 2.

cos 2A 1 2 sin2 A sin 2A 2 sin A cos A 2 tan A 1 tan2 A

Sketch a triangle in quadrant II and use it to nd sin : y sin 12 13 . sin 2 2 sin cos 12 2 13

tan 2A

13 12 5
x

5 120 13 169

Product-to-Sum Identities cos A cos B sin A sin B sin A cos B cos A sin B 1 cosA B cosA B 2 1 cosA B cosA B 2 1 sinA B sinA B 2 1 sinA B sinA B 2

Write sin sin 2 as the difference of two functions. sin sin 2 1 cos 2 cos 2 2 1 cos3 cos 2 1 1 cos3 cos 2 2 1 1 cos 3 cos 2 2

Sum-to-Product Identities sin A sin B 2 sin

sin A sin B 2 cos

cos A cos B 2 cos

cos A cos B 2 sin Half-Angle Identities cos tan A 2

AB AB cos 2 2 AB AB sin 2 2

AB AB cos 2 2

AB AB sin 2 2

Write cos cos 3 as a product of two functions. cos cos 3 2 cos 2 cos

2 cos 2 cos 2 cos 2 cos


3 3 cos 2 2
4 2 cos 2 2

A A sin A 1 cos A tan 2 1 cos A 2 1 cos A A 1 cos A The sign is chosen based on the tan 2 sin A quadrant of A 2 .

1 cos A 2

sin

A 2

Find the exact value of tan 67.5. We choose the last form with A 135. tan 67.5 tan
2 135 1 cos 135 1 2 2 2 sin 135 2

1 cos A 2

2 1 2 2 2

2 2 2 2 2

or 2 1

Rationalize the denominator; simplify.

LIALMC07_0321227638.QXP

2/26/04

10:48 AM

Page 678

678 CHAPTER 7 Trigonometric Identities and Equations

CONCEPTS

EXAMPLES

7.5 Inverse Circular Functions


Range Inverse Function y sin1 x y cos x y tan x y cot x
1 1 1 1

Evaluate y cos1 0. Write y cos1 0 as cos y 0. Then y 2 , because


1 cos 2 0 and 2 is in the range of cos x.

Domain 1, 1 1, 1 , ,

Interval
2, 2

Quadrants of the Unit Circle I and IV I and II

0,

2, 2

I and IV I and II I and II I and IV


y

0, 0, , y

y sec x , 1 1, y csc1 x , 1 1,
y 2

2, 2

, y

(1, 2)
0 x 1

( 1, )

Evaluate sin tan1 3 4 .


3 1 Let u tan1 3 4 . Then tan u 4 . Since tan x is negative in quadrant IV, sketch a triangle as shown.
y

1 2

(0, 2)
1 0

y = cos 1 x (1, 0)
1

(1, 2 )

y = sin 1 x
y 2

4 u 5

y=
2

tan 1 x
1

0 2

(1, 4) x
1 2

We want sin tan1 3 4 sin u. From the triangle, sin u 3 5.

1, 4

See page 649 for graphs of the other inverse circular (trigonometric) functions.
7.6 Trigonometric Equations

Solving a Trigonometric Equation 1. Decide whether the equation is linear or quadratic in form, so you can determine the solution method. 2. If only one trigonometric function is present, rst solve the equation for that function. 3. If more than one trigonometric function is present, rearrange the equation so that one side equals 0. Then try to factor and set each factor equal to 0 to solve. 4. If the equation is quadratic in form, but not factorable, use the quadratic formula. Check that solutions are in the desired interval. 5. Try using identities to change the form of the equation. It may be helpful to square both sides of the equation rst. If this is done, check for extraneous solutions.

Solve tan 3 23 over the interval 0, 360. Use a linear method. tan 3 23 tan 3

60
Another solution over 0, 360 is

60 180 240.
The solution set is 60, 240.

LIALMC07_0321227638.QXP

2/26/04

10:48 AM

Page 679

CHAPTER 7

Review Exercises 679

Chapter 7 Review Exercises


2. A 3. C 4. F 5. D cos2 6. E 7. 1 8. sin 1 cos 1 9. 10. cos2 sin 4 4 11. sin x ; tan x ; 5 3 4 3 cotx 12. cot x ; 4 5 41 41 csc x ; sec x 5 4 13. E 14. B 15. J 16. A 17. I 18. C 19. H 20. D 4 315 ; 21. G 22. B 23. 20 415 3 4 315 ; ;I 20 415 3 4 911 1211 3 ; ; 24. 50 50 4 911 1 ; IV 25. 2 1211 3 26. 6 3 1. B

Concept Check For each expression in Column I, choose the expression from Column II that completes an identity. I 1. sec x 3. tan x 5. tan2 x 2. csc x 4. cot x 6. sec2 x 1 A. sin x C. E. sin x cos x 1 cos2 x II B. D. F. 1 cos x 1 cot 2 x cos x sin x

Use identities to write each expression in terms of sin and cos , and simplify. 7. sec2 tan2 9. tan2 1 cot 2 8. cot sec

10. csc cot

11. Use the trigonometric identities to nd sin x, tan x, and cotx, given cos x 3 5 and x is in quadrant IV.
12. Given tan x 5 4 , where 2 x , use the trigonometric identities to nd cot x, csc x, and sec x.

Concept Check For each expression in Column I, use an identity to choose an expression from Column II with the same value. I 13. cos 210 15. tan35 17. cos 35 19. sin 75 21. cos 300 14. sin 35 16. sin 35 18. cos 75 20. sin 300 22. cos55 A. sin35 C. II

E. cos 150 cos 60 sin 150 sin 60 F. cot35 G. cos2 150 sin2 150 H. sin 15 cos 60 cos 15 sin 60 I. cos35 J. cot 125

B. cos 55 D. 2 sin 150 cos 150

1 cos 150 2

For each of the following, nd sinx y, cosx y, tanx y, and the quadrant of x y. 23. sin x 24. sin x 1 4 , cos y , x and y in quadrant III 4 5

1 4 , cos y , x in quadrant I, y in quadrant IV 10 5

Find each of the following.

1 , given cos , with 90 180 2 2 1 26. sin y, given cos 2y , with y 3 2


25. cos

LIALMC07_0321227638.QXP

2/26/04

10:48 AM

Page 680

680 CHAPTER 7 Trigonometric Identities and Equations sin 2x sin x x cot cos 2x cos x 2 1 cos 2x tan x sin 2x 2 46. 47. 4 3 3 3 49. 50. 2 4 6 2 3 52. 53. 3 3 4 60 55. 60 56. 0 60.67924514 41.33444556 36.4895081 12.51631252 73.26220613 7.673567973 3 3 1 64. 65. 2 4 7 68. 0 69. 67. 4 4 9 10 3 71. 72. 10 2 7 294 1256 92 1 1 u2 75. u 3 , 2 2

27. 28. 45. 48. 51. 54. 57. 58. 59. 60. 61. 62. 63. 66. 70. 73. 74. 76.

Graph each expression and use the graph to conjecture an identity. Then verify your conjecture algebraically. 27. sin 2x sin x cos 2x cos x 28. 1 cos 2x sin 2x

Verify that each equation is an identity. 29. sin2 x sin2 y cos2 y cos2 x 31. x sin2 x cos2 2 2 cos x 2 tan A csc A 30. 2 cos3 x cos x 32. 34. 36. 2 sin 2x sin x sec x 2 tan B sec2 B sin 2B 2 cot x csc2 x 2 tan 2x 1 tan2 1 tan2 cos2 x sin2 x sec x

33. 2 cos A sec A cos A 35. 1 tan2 2 tan csc 2 37. tan sin 2 2 2 cos2 39. 2 tan x csc 2x tan2 x 1 41. tan cos2

38. csc A sin 2A sec A cos 2A sec A 40. 2 cos2 1 42. sec2 1

2 tan cos2 tan 1 tan2 cos2 x sin2 x sec x

sec 2 1 sec 2 1

43. 2 cos3 x cos x

44. sin3 sin cos2 sin

Give the exact real number value of y. Do not use a calculator. 45. y sin1 2 2 46. y arccos 49. y cos1 52. y arccsc

77. .463647609, 3.605240263

48. y arcsin1 51. y sec12


1 2 2 2 23 3 3 2

47. y tan1 3 50. y arctan 3 3

53. y arccot1

Give the degree measure of . Do not use a calculator. 54. arccos 1 2 55. arcsin

56. tan1 0

Use a calculator to give the degree measure of . 57. arctan 1.7804675 60. cot 4.5046388
1

58. sin1.66045320 61. arcsec 3.4723155

59. cos1 .80396577 62. csc1 7.4890096

Evaluate the following without using a calculator. 63. cosarccos1 66. arcsecsec 69. sin arccos 64. sin arcsin 67. tan1 tan

3 4


3 2

65. arccos cos


3 4

68. cos1cos 0 71. coscsc12

70. cosarctan 3

LIALMC07_0321227638.QXP

2/26/04

10:48 AM

Page 681

CHAPTER 7

Review Exercises 681

78.

.7297276562,

2.411864997 79. 80.

3 5 7 , , , 4 4 4 4

, 2

72. sec 2 sin1


1 3

73. tan arcsin

3 5 arccos 5 7

Write each of the following as an algebraic (nontrigonometric) expression in u. 74. cos arctan

3 5 7 9 11 , , , , , , 8 8 8 8 8 8

1 u2

75. tan arcsec

u2 1 u

13 15 , 8 8

7 9 11 13 15 , , , , 8 8 8 8 8 82. 0 2n, where n is any integer 83. 2n, 2n, 3 5 2n, where n is any integer 3 2n, 84. 2n, 6 3 4 7 2n, 2n, where n 6 3 is any integer 85. {270} 86. {45, 153.4, 225, 333.4} 87. {45, 90, 225, 270} 88. {15, 75, 195, 255} 89. {70.5, 180, 289.5} 90. {53.5, 118.4, 233.5, 298.4} 91. x arcsin 2y y 92. x 2 arccos 3 93. x


81.

3 5 , , , 8 8 8

Solve each equation for solutions over the interval 0, 2. 76. sin2 x 1 79. tan x cot x 77. 2 tan x 1 0 80. sec 2x 4
4

78. 3 sin2 x 5 sin x 2 0 81. tan2 2x 1 0

Give all solutions for each equation. 82. sec x x cos 2 2 83. cos 2x cos x 0 84. 4 sin x cos x 3

Solve each equation for solutions over the interval 0, 360. If necessary, express solutions to the nearest tenth of a degree. 85. sin2 3 sin 2 0 87. sin 2 cos 2 1 89. 3 cos2 2 cos 1 0 86. 2 tan2 tan 1 88. 2 sin 2 1 90. 5 cot 2 cot 2 0

Solve each equation in Exercises 91 97 for x. 91. 4y 2 sin x 94. 5y 4 sin x 3 97. arccos x arctan 1 92. y 3 cos 95. 11 12 x 2 93. 2y tan3x 2 96. arccos x arcsin 2 7

1 2 arctan 2y 3 3 5y 3 4

94. x arcsin 95. 0 98. t 96.

50 d 550 arccos 450


35 7 97. 1 2

4 x arctan 3 2

98. Solve d 550 450 cos

t for t in terms of d. 50

(Modeling) Solve each problem. 99. Viewing Angle of an Observer A 10-ft-wide chalkboard is situated 5 ft from the left wall of a classroom. See the gure. A student sitting next to the wall x feet from the front of the classroom has a viewing angle of radians. (a) Show that the value of is given by the function dened by f x arctan
5 10

99. (b) 8.6602567 ft; There may be a discrepancy in the nal digits. 5 f ( x ) = arctan 15 x arctan x
1

( )

()
20

15 5 arctan . x x

(b) Graph f x with a graphing calculator to estimate the value of x that maximizes the viewing angle.

LIALMC07_0321227638.QXP

2/26/04

10:48 AM

Page 682

682 CHAPTER 7 Trigonometric Identities and Equations 100. 48.8 101. The light beam is completely underwater. 102. (a) 42.2 (b) 90 (c) 48.0 1 103. y = csc x
2

100. Snells Law Recall Snells law from Exercises 102 and 103 of Section 5.3: c 1 sin 1 , c 2 sin 2
Air 2

2 Radian mode

where c 1 is the speed of light in one medium, Water c 2 is the speed of light in a second medium, 1 and 1 and 2 are the angles shown in the gure. Suppose a light is shining up through water into the air as in the gure. As 1 increases, 2 approaches 90, at which point c1 no light will emerge from the water. Assume the ratio c2 in this case is .752. For what value of 1 does 2 90? This value of 1 is called the critical angle for water. 101. Snells Law Refer to Exercise 100. What happens when 1 is greater than the critical angle?

104. (a), (b)


2 1 1

102. British Nautical Mile The British nautical mile is dened as the length of a minute of arc of a meridian. Since Earth is at at its poles, the nautical mile, in feet, is given by L 6077 31 cos 2,

A nautical mile is the length on any of the meridians cut by a central angle of measure 1 minute.

In both cases, sin1 .4 .41151685.

where is the latitude in degrees. See the gure. (Source: Bushaw, Donald et al., A Sourcebook of Applications of School Mathematics. Copyright 1980 by The Mathematical Association of America.) (a) Find the latitude between 0 and 90 at which the nautical mile is 6074 ft. (b) At what latitude between 0 and 180 is the nautical mile 6108 ft? (c) In the United States, the nautical mile is dened everywhere as 6080.2 ft. At what latitude between 0 and 90 does this agree with the British nautical mile? 103. The function y csc1 x is not found on graphing calculators. However, with some models it can be graphed as y x 0 x 0

tan1 x 2 1 . 2

(This formula appears as Y1 in the screen here.) Use the formula to obtain the graph of y csc1 x in the window 4, 4 by 2 , 2 . 104. (a) Use the graph of y sin1 x to approximate sin1 .4. (b) Use the inverse sine key of a graphing calculator to approximate sin1 .4.

Chapter 7 Test
1. sin x 561 ; 61 661 cos x 2. 1 61
3 1. Given tan x 5 6 , 2 x 2, use trigonometric identities to nd sin x and cos x.

2. Express tan2 x sec2 x in terms of sin x and cos x, and simplify.


2 3. Find sinx y, cosx y, and tanx y, if sin x 1 3 , cos y 5 , x is in quadrant III, and y is in quadrant II.

4. Use a half-angle identity to nd sin22.5.

LIALMC07_0321227638.QXP

2/26/04

10:48 AM

Page 683

CHAPTER 7 Test

683

2 242 ; 15 42 21 cosx y ; 15 22 421 tanx y 8 42 2 2 4. 2 5. sec x sin x tan x cos x x 6. cot cot x csc x 2 9. (a) sin (b) sin t 10. (a) V 163 cos 2 1 (b) 163 volts; sec 240 3. sinx y

Graph each expression and use the graph to conjecture an identity. Then verify your conjecture algebraically. 5. sec x sin x tan x 6. cot x cot x 2

Verify that each equation is an identity. 7. sec2 B 1 1 sin2 B 8. cos 2A cot A tan A csc A sec A

9. Use an identity to write each expression as a trigonometric function of alone. (a) cos270 (b) sin 10. Voltage The voltage in common household current is expressed as V 163 sin t, where is the angular speed (in radians per second) of the generator at the electrical plant and t is time (in seconds). (a) Use an identity to express V in terms of cosine. (b) If 120, what is the maximum voltage? Give the smallest positive value of t when the maximum voltage occurs. 11. Graph y sin1 x, and indicate the coordinates of three points on the graph. Give the domain and range.

11. 1, 1; , 2 2
y 2 (0, 0) 1

(1, 2)
y = sin
1

x x

12. Find the exact value of y for each equation. (a) y arccos

(1, 2)
12. (a) (c) 13. 14. 16. 17. 18. 2 3

(b)

(c) y tan1 0


1 2 2 3

(b) y sin1

(d) y arcsec2


3 2 1 3

2 0 (d) 3 42 5 (a) (b) 3 9 u1 u2 15. {90, 270} 1 u2 {18.4, 135, 198.4, 315}

13. Find each exact value. (a) cos arcsin

(b) sin 2 cos1

14. Write tanarcsin u as an algebraic (nontrigonometric) expression in u. 15. Solve sin2 cos2 1 for solutions over the interval 0, 360. 16. Solve csc2 2 cot 4 for solutions over the interval 0, 360. Express approximate solutions to the nearest tenth of a degree. 17. Solve cos x cos 2x for solutions over the interval 0, 2. 18. Solve 23 sin x 3, giving all solutions in radians. 2 4 3

0,

2 4 , 3 3

4 2 4n, 4n, where 3 3 n is any integer 1 19. (a) x arccos y 3 4 (b) 5

19. Solve each equation for x. (a) y cos 3x (b) arcsin x arctan

20.

5 11 17 sec, sec, sec 6 6 6

20. (Modeling) Movement of a Runners Arm A runners arm swings rhythmically according to the model y

1 cos t 8 3

where y represents the angle between the actual position of the upper arm and the downward vertical position and t represents time in seconds. At what times over the interval 0, 3 is the angle y equal to 0?

LIALMC07_0321227638.QXP

2/26/04

10:48 AM

Page 684

684 CHAPTER 7 Trigonometric Identities and Equations

Chapter 7 Quantitative Reasoning


How can we determine the amount of oil in a submerged storage tank?
The level of oil in a storage tank buried in the ground can be found in much the same way a dipstick is used to determine the oil level in an automobile crankcase. The person in the gure on the left has lowered a calibrated rod into an oil storage tank. When the rod is removed, the reading on the rod can be used with the dimensions of the storage tank to calculate the amount of oil in the tank. Suppose the ends of the cylindrical storage tank in the gure are circles of radius 3 ft and the cylinder is 20 ft long. Determine the volume of oil in the tank if the rod shows a depth of 2 ft. (Hint: The volume will be 20 times the area of the shaded segment of the circle shown in the gure on the right.)

3 ft 2 ft

20 9 arctan 8 8 165 ft3

You might also like